Anda di halaman 1dari 57

SOAL USBN BAHASA INDONESIA PAKET 1

KELAS XII SMK

A.Pilihlah jawaban yang paling tepat!


1. Bacalah teks berikut!
Industri komponen otomotif Indonesia belum mampu bersaing dengan industri komponen luar
negeri. Kondisi ini terjadi karena kurangnya kepercayaan masyarakat pengguna komponen
otomotif dalam negeri. Kita masih terhambat dalam pengembangan teknologi. Kita belum bisa
menyamai posisi Singapura, Vietnam, dan India yang telah mengembangkan berbagai teknologi.
Kelemahan otomotif Indonesia ialah pada teknologi mesin dan pengembangankapasitas (capasity
building) sehingga hanya masuk sebagai negara resembling dan adopting, berbeda dengan
Thailand dan Malaysia yang sudah menjadi basis produksi.
Gagasan pokok teks tersebut adalah ... .
A. negara berbasis produksi industri
B. persaingan industri komponen otomatif
C. hambatan pengembangan industri otomotif
D. negara resembling dan adopting
E. keunggulan industri otomotif Indonesia

2. Bacalah teks berikut!


Googling telah menjadi istilah umum dan paling banyak dipakai masyarakat untuk mencari data
atau informasi apapun di dunia saat ini. Panggilan akrab ‘Mbah Google’ yang mengacu pada
kemampuan mesin pencari (searchengine) itu, telah lama disematkan pada pemakai gadget di
Indonesia. Sebagai negara dengan pengguna internet dan populasi ponsel pintar terbanyak,
Indonesia menjadi pasar fenomenal yang potensial, sekaligus area penting bagi pengembangan
layanan google.
Makna istilah disematkan dan fenomenal pada teks tersebut adalah ... .
A. dialamatkan, trend
B. disampaikan, terbesar
C. ditambatkan, terkenal
D. disebutkan, terbaik
E. ditujukan, strategis

3. Bacalah teks berikut!


Ada tiga hal yang harus dilakukan orang tua dalam menjaga tumbuh kembang anak-anak.
Pertama, orangtua harus menjadi pusat komunikasi dengan anak yang lebih mendalam. Saat
inibanyak orangtua yang tidak mau mendengar curhat anak sehingga anak lebih memilih untuk
curhat kepada orang lain yang terkadang kurang tepat. Kedua, orangtua harus menunjukkan
kepedulian lebih tinggi dan tidak mengandalkan pihak ketiga seperti sekolah dan pengasuh.
Ketiga, orangtua harus mengetahui psikologi anak. Orangtua jangan memaksakan mendidik
dengan teknik yang digunakan 10 atau 20 tahun yang lalu.
Simpulan teks tersebut adalah ... .
A. Komunikasi antara anak dan orangtua sangat penting dalam menjaga tumbuh kembang anak.
B. Kepedulian orangtua harus ditunjukkan pada anak agar mereka mengerti adanya kasih sayang.
C. Psikologi anak harus dipahami orangtua untuk lebih memahami psikologi dan dunia anak.
D.
E. Penggunaan teknik mendidik masa kini sangat diperlukan karena paling sesuai dengan kondisi
anak.

4. Bacalah teks berikut!


Dalam rangka memperingati Hari Kebangkitan Nasional, salah satu komunitas lari di Jakarta
mengadakan kegiatan lari santai dengan tema Lari Kebangkitan Indonesia Bangkit. Tercatat 600
pelari dari 26 komunitas yang ada di Jakarta dan sekitarnya mengikuti ajang yang menempuh
jarak 5 kilometer. Kegiatan ini mendorong orang-orang yang hobi lari untuk semakin
bersemangat. Ini adalah bukti nyata bahwa setiap komunitas melakukan hal yang produktif.
Kegiatan ini digelar sebagai ajang untuk berkumpulnya komunitas lari dan mempromosikan gaya
hidup sehat, serta menginspirasi orang-orang untuk membangun Indonesia yang lebih baik.

Pernyataan yang sesuai dengan isi teks tersebut adalah ....


A. Promosi dan inspirasi gaya hidup sehat diperoleh pada kegiatan lari santai.
B. Kegiatan lari santai mengundang pelari dari beberapa komunitas lari.
C. Ajang berkumpulnya para pelari yang memiliki gaya hidup sehat dan santai.
D. Kegiatan ini digelar untuk mewadahi kebangkitan Indonesia saat ini.
E. Di Jakarta saat ini terdapat 600 pelari dan 26 komunitas lari yang produktif.

5. Bacalah teks berikut!


Kecintaannya pada batik menyebabkan ia senang berbelanja di pasar Bringhardjo. Terlebih
ketika ia berhasil memadu padankan dua motif yang berbeda. Kedua motif ini dapat dipadu dan
dikreasi menjadi sebuah busana yang menyenangkan pemakainya. Kreasi-kreasi ini yang dicari
penggemar batik saat ini.
Kata yang penulisannya tidak sesuai dengan ejaan bahasa Indonesai adalah ... .
A. di pasar
B. Bringhardjo
C. memadu padankan
D. dipadu
E. dikreasi

6. Bacalah teks berikut!


Gagasan utama : Manfaat gerakan literasi sekolah bagi siswa
Gagasan penjelas :
(1) Membiasakan siswa gemar membaca;
(2) Memotivasi siswa untuk lebih literat;
(3) Membaca buku 15 menit sebelum jam pertama setiap hari;
(4) Mendorong siswa berpikir dan bertindak secara kritis;
(5) Menuliskan hasil bacaan setiap minggu.
Gagasan penjelas yang tepat untuk gagasan utama tersebut adalah ... .
A. (1), (2), (3)
B. (1), (3), (4)
C. (1), (2), (4)
D. (2), (3), (5)
E. (2), (4), (5)

7. Cermati paragraf berikut ini!


Kandungan gizi susu kambing tidak kalah dengan susu sapi. Hanya sebagian kecil
masyarakat mengonsumsi susu kambing. Kurangnya minat masyarakat mengonsumsi susu
kambing karena ada anggapan bahwa susu kambing tidak boleh dikonsumsi penderita tekanan
darah tinggi. Mitos tersebut tidak benar. Kandungan kalium dalam susu kambing justru
berfungsi menstabilkan tingginya tekanan darah, mengalirkan fungsi kerja jantung, dan
menekan risiko terkena arteriosklerosis.
Tujuan penulis sesuai isi paragraf di atas adalah …
A. Menginformasikan bahwa susu kambing bermanfaat bagi manusia.
B. Mengharapkan konsumen untuk mengonsumsi susu sapi.
C. Menjelaskan bahwa mitos susu kambing benar.
D. Mengajak penderita tekanan darah tinggi mengonsumsi susu kambing.
E. Menjelaskan kerugian apabila manusia megonsumsi susu kambing.

8. Bacalah teks di bawah ini!


Pada rute terakhir itu, kondisi kami sudah tidak optimal. Padahal jalan yang akan kami
telusuri sungguh berat dengan… tinggi. Lorong gua menurun terjal dengan jarak 60 meter
sampai 70 meter. Bukan… licin, air yang tergenang juga mengalir dan… mengikuti bentuk
lorong gua. Kalau sampai terpeleset bisa berbahaya.
Kata baku yang tepat untuk melengkapi paragraf tersebut adalah....
A. risiko, hanya, jatuh
B. risiko, cuman, jatuh
C. resiko, cuman, jatuh
D. resiko, hanya, jatoh
E. resiko, cuman, jatuh

9. Bacalah teks berikut!


Pada suatu hari seorang guru di sebuah sekolah dasar sedang bertanya kepada seorang muridnya
tentang hasil belajar menghafalkan huruf.
Pak guru : “Bobi, sudah berapa huruf yang Bobi hafal?
Bobi “ Bobi hafal C D E F G A B C”. Bobi menyebutkan dengan lancar.
(Mendengar jawaban dari Bobi, pak guru pun bingung lalu bertanya kembali
kepada Bobi).
Pak guru : “Kenapa Bobi hanya menghafalkan tujuh huruf saja?”
Bobi (menjawab dengan lantang) “Menghafal tujuh huruf saja, Bobi bisa jadi pemusik
yang hebat dan menghasilkan banyak uang”.
Pak guru : ”Benar. Benar”. Pak guru pun mengangguk-ngangguk saja mendengar
jawaban Bobi.

Maksud teks tersebut adalah ... .


A. Seorang siswa mampu berpikir lebih jauh dari apa yang kita ajarkan.
B. Seorang pemusik tidak harus menghafal seluruh huruf yang ada.
C. Seorang guru selalu mengajarkan sesuai aturan yang berlaku.
D. Serangkaian huruf yang dihafal lebih penting daripada sejumlah uang.
E. Sebuah jawaban yang diberikan siswa tidak dapat diduga sebelumnya.

10. Bacalah teks berikut!


Bunga sepatu atau kembang sepatu merupakan tanaman yang dikenal mempunyai bentuk bunga
yang indah dan berwarna-warni. Di Indonesia, pohon bunga sepatu banyak ditanam sebagai
tanaman hias dan ada pula yang menggunakannya sebagai pagar hidup. Secara alamiah, bunga
sepatu mempunyai nama latin Hibiscus Rosa Sinensis yang digolongkan dalam tumbuhan semak.
Sinonim kata bentuk dan semak pada teks tersebut adalah ... .
A. jenis, rendah
B. model, perdu
C. ciri, pendek
D. jenis, perdu
E. model, rendah
11. Bacalah teks berikut!
Bu Ambar : Sya, besok kita kumpul di aula pukul 10.00 untuk pengarahan.
Rhaisya : Saya izin, ya, Bu. Saya ada technical meeting lomba pukul 08.00 s.d. 12.00.
Bu Ambar : Kamu izin saja ke panitia.
Rhaisya : Saya ketua, Bu. Jadi saya harus ikut.
Bu Ambar : Kamu penjab juga, kan?
Rhaisya : Ya, Bu. Bagaimana kalau pengarahannya pukul 13.00. Jadi, saya bisa hadir.
Bu Ambar : Bagus. Ide yang bagus. Terima kasih idenya, Sya.
Rhaisya : Iya, Bu. Sama-sama.

Isi negosiasi tersebut adalah ... .


A. kesepakatan penundaan waktu pengarahan kegiatan
B. perbedaan pendapat waktu technical meeting
C. kesepakatan penundaan waktu technical meeting
D. pemilihan penanggungjawab kegiatan
E. permohonan izin mengikuti pengarahan kegiatan

12. Bacalah kedua teks berikut!


Teks 1 Teks 2
Gaya belajar anak merupakan cara anak dalam Sekolah adalah salah satu sarana untuk
menerima, memproses, memahami, dan membantu anak untuk tumbuh optimal. Untuk
menyimpan informasi yang diterima dalam itu orang tua harus selektif memilih sekolah.
belajar di sekolah. Setiap anak memiliki gaya Orangtua perlu melakukan observasi sehingga
belajar yang berbeda. Secara umum ada 3 dapat membandingkan beberapa sekolah
macam gaya belajar, yaitu visual/melihat, sebelum memutuskan sekolah yang cocok
auditory/mendengar, dan kinestetik/gerakan. untuk anak mereka. Pilihlah sekolah dengan
Perbedaan ini mengharuskan orang tua dan tenaga pengajar yang peduli anak, program
guru menggunakan cara mengajar yang kurikulum yang membuat anak berkarakter,
berbeda agar anak dapat berkembang secara dan jarak yang tidak terlalu jauh dari tempat
maksimal. tinggal.
Informasi yang terdapat pada kedua teks tersebut adalah ... .
A. Kedua teks berisi informasi tentang pembelajaran di sekolah dan kurikulum yang digunakan.
B. Kedua teks membicarakan kriteria sekolah yang baik dan cara belajar yang sesuai dengan
kebiasaan anak.
C. Teks 1 berisi informasi kepedulian pendidik, teks 2 berisi informasi sarana pembelajaran di
sekolah.
D. Teks 1 berisi informasi tentang cara belajar anak, teks 2 berisi informasi tentang cara memilih
sekolah bagi anak.
E. Teks 1 berisi sekolah berkarakter, teks 2 berisi proses penyimpanan informasi belajar.

13. Bacalah teks berikut!


Makam ini terletak di kawasan Pasar Tomak, tak jauh dari dermaga. Areal ini ramai dikunjungi
pada saat-saat tertentu. Biasanya hal pertama yang dilakukan para pengunjung adalah
mengunjungi makam orang tua atau kerabat lainnya. Di sana kita juga bisa menyewa pemandu
tur untuk menjelaskan berbagai tempat wisata [ ....] mengunjungi makam. Kunjungan diakhiri
dengan berbelanja berbagai cenderamata khas setempat.
Konjungsi yang tepat untuk melengkapi teks tersebut adalah ... .
A. Sebelum D. dan
B. Setelah E. walaupun
C. ketika
14. Bacalah teks berikut!
Berbicara tentang garansi, apalagi garansi mesin, bukan hal gampang. Lebih gampang kita bicara
tentang garansi terhadap sebuah barang yang memang tidak bisa rusak. Garansi untuk mesin itu
sebenarnya meliputi jaminan[....], kemudahan mendapat spare- part mesin, memperoleh
informasi yang jelas sebelum membeli, hingga pelatihan terhadap operator mesin sampai bisa.
Padahal, tidak banyak penjual mesin yang memberikan garansi selengkap itu.
Kata bentukan yang tepat untuk melengkapi teks tersebut adalah ... .
A. perbaikan
B. perbaiki
C. pembaikan
D. memperbaiki
E. diperbaiki

15. Bacalah teks berikut!


(1) Perhatian masyarakat dunia terhadap persoalan pertanian, kesehatan, dan lingkungan global
terakhir ini semakin meningkat. (2) Kepedulian tersebut ditunjukkan dengan usaha-usaha yang
konkret untuk menghasilkan pangan tanpa menyebabkan terjadinya kerusakan sumber daya alam.
(3) Salah satu usaha yang dirintis adalah pengembangan pertanian organik. (4) Pertanian jenis ini
akrab dengan lingkungan dan menghasilkan pangan yang sehat. (5) Untuk memperoleh hasil
yang maksimal, residu obat-obatan dan zat-zat kimia masih tetap digunakan.
Kalimat penjelas yang sumbang dalam teks tersebut ditandai dengan nomor ... .
A. (1)
B. (2)
C. (3)
D. (4)
E. (5)

16. Bacalah teks berikut!


Jumlah timbunan sampah per hari diperkirakan 175.000 – 176. 000 ton. Jumlah tersebut akan
meningkat seiring dengan meningkatnya jumlah penduduk. Dengan jumlah itu, sekitar 14%nya
adalah sampah plastik atau mencapai 9 ton per tahun. Sampah plastik itu menambah kompleks
permasalah sampah karena juga sangat berpotensi mencemari lautan termasuk ekosistem di
dalamnya. Untuk mengatasi masalah sampah pemerintah melalui Kementerian LHK berupaya
mengubah paradigma pengolahan sampah. [ .... ] Kini, pengelolaan sampah akan diubah menjadi
diolah di sumber. Pengelolaan sampah cara baru tersebut diharapkan bisa menjadi energi dan
memiliki nilai jual.
Kalimat yang tepat untuk melengkapi teks tersebut adalah ... .
A. Saat ini sampah dikelola dengan berbagai cara yang disesuaikan dengan lokasi sampah.
B. Dewasa ini pengelolaan sampah penduduk dilakukan dengan dengan ditimbun atau dibakar.
C. Sampah yang dikelola dengan paradigma lama akan selalu menimbulkan polusi udara.
D. Sebelumnya, pengelolaan sampah dari tahapan kumpul, angkut, dan buang atau kumpul,
angkut, dan kelola (manage).
E. Paradigma yang digunakan saat ini adalah kumpul, angkut, buang, dan timbun di TPA.

17. Cermati teks pantun berikut!


(1) Kita juga punya borobudur
(2) Wujudkan dalam laku dan tutur
(3) Indonesia punya kain batik
(4) Tunjukkanlah sikap yang terbaik
Urutan teks pantun yang tepat adalah ... .
A. (3), (4), (1), (2)
B. (3), (2), (4), (1)
C. (3), (1), (4), (2)
D. (1), (4), (2), (3)
E. (1), (3), (2), (4)

18. Cermati teks pantun berikut!


(1) Di Jakarta ada tugu monas
(2) [ ...]
(3) Kami bangga padamu timnas
(4) Yang ‘tlah berjuang dengan tepat
Larik yang tepat untuk melengkapi teks pantun tersebut adalah ... .
A. Ukuran tugu tiga satu empat
B. Ornamennya belah ketupat
C. Penduduknya sangatlah rapat
D. Di Papua ada Raja ampat
E. Panjangnya terukur tepat

19. Cermati penulisan alamat surat berikut!


Yth. Sdr. Bachtiar Saleh, SE.
Kepala Bagian Personalia PT Indocool
Jalan Pesona Alam Raya
Bogor
Kesalahan penggunaan tanda baca pada penulisan alamat surat tersebut adalah ... .
A. Yth.
B. Sdr.
C. SE.
D. PT
E. Jalan

20. Bacalah kutipan cerita pendek berikut!


Bungai ingin seperti Kak Putir, bisa menganyam tikar...tapi Kak Putir tak mau mengajari. Bungai
ingin seperti Kak Putir, punya banyak teman. Bungai ingin ikut bermain bersama, tapi Bungai
hanya boleh melihat saja. Diam-diam Bungai mengikuti kak Putir ke sanggar. Bungai meniru
gerakan Kak Putir menari. “Bungai! Anak laki-laki tak boleh menari tarian Bahalai,” kata Kak
Putir. Bungai melihat dan menirukan orang menari Mandau. Hore! Ternyata ada tarian yang
cocok untuk Bungai. Bungai dan Kak Putir menari bersama. Mereka bisa menari Manasai
bersama-sama.

Unsur ekstrinsik yang terdapat dalam kutipan cerita pendek tersebut adalah ... .
A. sosial
B. agama
C. politik
D. pendidikan
E. moral
21. Bacalah teks berikut!
Selama ini kita mengetahui bahwa bahan utama pembuatan kertas adalah kayu. Namun sekarang,
kayu bukan satu-satunya bahan utama pembuatan kertas tersebut. Limbah tapioka [ ...]
menggantikan kayu sebagai bahan utama kertas. Hal ini merupakan hasil penelitian Pusat
Penelitian Sumber Daya Hayati dan Bioteknologi IPB pada 2009-2010 tentang tapioka.
Istilah yang tepat untuk melengkapi teks tersebut adalah ... .
A. berguna
B. bermanfaat
C. berpotensi
D. bersaing
E. berhasil

22. Bacalah teks ulasan berikut!


Makmur Makka selalu menulis apa adanya secara obyektif. Dalam biografi ini ia menulis betapa
hati Habibie yang selalu mendidih jika melihat kesenjangan dalam masyarakat yang tak kunjung
selesai. Mengenang saat-saat yang tidak pernah dilupakannya, Habibie mengatakan, “Kemauan
keras saya dan perkenan Tuhan menyebabkan masa kritis itu dapat saya lalui dengan baik.”
Mengenai pribadinya, Makka menulis, “tidak berlebihan untuk mengatakan bahwa sampai saat
ini tetap penuh rasa simpatik. Ia disiplin, berjiwa besar, punya solidaritas dan loyalitas tinggi,
ilmuwan yang beriman, dan berwatak teguh.”
Isi teks ulasan tersebut adalah ... .
A. keunggulan buku
B. kelemahan buku
C. sinopsis/isi buku
D. keunikan buku
E. deskripsi buku

23. Bacalah teks berikut!


(1) Pelatihan pengemasan ini bertujuan untuk meningkatkan pengetahuan para pelaku UKMK
dan masyarakat. (2) Pelatihan ini difasilitasi oleh Bidang Perindustrian. (3) Para peserta pelatihan
dipersiapkan untuk dapat bersaing dalam pasar global, khususnya dalam hal menciptakan strategi
pemasaran yang baik. (4) Materi yang diberikan meliputi dasar pengemasan, sejarah
pengemasan, hingga cara mendaftarkan nomor barcode. (5) Pemerintah berharap, mereka dapat
mengemas produk agar aman dan bernilai jual tinggi.
Kalimat simpleks (kalimat tunggal) dalam teks tersebut ditandai dengan nomor ... .
A. (1)
B. (2)
C. (3)
D. (4)
E. (5)

24. Bacalah kalimat berikut!


Pemerintah telah meresmikan Kampung Buruh Migran (KBM) di Wonosobo sebagai proyek
percontohan komunitas buruh migran di berbagai daerah.
Kalimat lain yang semakna dengan kalimat tersebut adalah ... .
A. Pemerintah menyediakan Kampung Buruh Migran (KBM) di Wonosobo sebagai proyek
percontohan buruh migran di berbagai daerah.
B. Pemerintah menempatkan Kampung Buruh Migran (KBM) di Wonosobo sebagai proyek
komunitas buruh migran di berbagai daerah.
C. Kampung Buruh Migran (KBM) di Wonosobo telah dimanfaatkan sebagai proyek komunitas
buruh di berbagai daerah.
D. Pemerintah telah menjadikan Kampung Buruh Migran (KBM) di Wonosobo sebagai proyek
percontohan komunitas buruh migran di berbagai daerah.
E. Pemerintah telah menyelesaikan keberadaan Kampung Buruh Migran (KBM) di Wonosobo
sebagai proyek percontohan berbagai daerah.

25. Bacalah teks berikut!


Kurangi tumpukan sampah dengan berbagai cara pengolahan di tempat.
Inti kalimat yang tepat untuk kalimat tersebut adalah ... .
A. kurangi sampah
B. olah sampah
C. cara mengolah sampah
D. olah di tempat
E. tumpukan sampah

26. Bacalah kutipan puisi berikut!


Bung Karno
Pacu kuda jihadmu
Jangan mundur lagi
Kami turunan Iskandar Muda
Tetesan darah Ratu Safiah
Anak cucu mujahid Tiro
Kemenakan Umar Pahlawan
Telah siap bertempur
Kami sedang menggempur
Ali Hasjmy
Tema puisi tersebut adalah ... .
A. kesiapan
B. kepatuhan
C. kekuatan
D. kemarahan
E. keberanian

27. Bacalah kutipan novel berikut!


“Ini yang paling penting. Kau kuberi amanah mengoperasikan alat ini,” Paman mengusap-usap
blender itu.
“Pasang telinga lambingmu itu baik-baik. Alat ini adalah teknologi dapur yang canggih. Baru
datang dari Jakarta dan telah lama kupesan dari A Tun. Harganya sangat mahal. Hanya rumah-
rumah menteri dan warung kopi terkenal di Jakarta yang bisa punya alat ini. Perlakukan ia
dengan penuh sopan santun! Hanya dalam keadaan darurat kau boleh memakainya.”
Andrea Hirata: Cinta di Dalam Gelas
Kalimat yang menunjukkan bukti ketegasan watak Paman pada kutipan novel tersebut adalah ... .
A. Kau kuberi amanah mengoperasikan alat ini.
B. Hanya dalam keadaan darurat kau boleh memakainya.
C. Alat ini adalah teknologi dapur yang canggih.
D. Baru datang dari Jakarta dan telah lama kupesan dari A Tun.
E. Harganya sangat mahal.
28. Bacalah teks berikut!
Meskipun belum banyak dikenal pembaca, ia tetap menuangkan ide-idenya yang inspiratif. Di
beberapa galeri dan sudut-sudut baca, kita dapat menikmati buah pena dan karya lainnya.
Kemampuan memadukan karya itulah yang terkadang kita belum memikirkannya.

Makna ungkapan buah pena yang terdapat dalam teks tersebut adalah ... .
A. lukisan
B. tulisan
C. gagasan
D. rancangan
E. pahatan

29. Bacalah teks berikut!


Tantangan bangsa Indonesia di era modern ini bukan lagi buta aksara. Saat ini [...] ditantang oleh
permasalahan kemiskinan struktural, ketidakpedulian sosial, dan ketidakmampuan memilah dan
mengkritisi informasi yang baik. Mengembangkan literasi lokal dalam konteks krisis lingkungan,
misalnya, dapat dilakukan dengan membuat cerita tentang anak-anakyang pergi ke sekolah dan
menggunakan masker karena daerah [...] sudah terkena kabut asap.
Kata ganti yang tepat untuk melengkapi teks tersebut adalah ... .
A. kami, mereka
B. kita, kami
C. kami, kita
D. mereka, kita
E. kita, mereka

30. Bacalah teks berikut!


Penelitian terbaru mengungkapkan anak-anak yang hidup di bawah garis kemiskinan memiliki
risiko tinggi terkena kanker di kemudian hari. Dalam studi itu, tim menganalisa 22 studi berbeda
dalam rangka menemukan hubungan antara risiko kanker dan gaya hidup masyarakat sejak masa
kecil. Secara luas, peneliti juga mengamati kondisi hidup yang buruk berperan meningkatkan
seluruh risiko kematian pada manusia.
Kata tidak baku dalam teks tersebut adalah ... .
A. menganalisa
B. risiko
C. tim
D. masa
E. kanker

31. Bacalah kalimat berikut!


Indonesia sendiri kini sudah menerapkan berbagai cara demi untuk menjadi bangsa berkarakter.
Perbaikan kalimat tidak efektif tersebut adalah ... .
A. Indonesia sendiri kini sudah menerapkan berbagai cara untuk menjadi bangsa berkarakter.
B. Indonesia kini sudah menerapkan berbagai cara demi untuk menjadi bangsa berkarakter.
C. Indonesia sudah menerapkan berbagai cara demi untuk menjadi bangsa berkarakter.
D. Indonesia kini sudah menerapkan berbagai cara untuk menjadi bangsa berkarakter.
E. Indonesia sendiri kini sudah menerapkan cara untuk menjadi bangsa berkarakter.
32. Bacalah teks berikut!
(1) Kementerian Pendidikan dan Kebudayaan mencatat dari 700 ribu guru kelompokbermain,
21% dan taman kanak-kanak, 49% hanya berijazah SMA/SMP.
(2) Selain itu, tenaga-tenaga pendidiknya pun masih banyak yang belum kompeten.
(3) Saat ini Lembaga Pendidikan Anak Usia Dini (PAUD) sudah menjangkau ke desa-desa.
(4) Sisanya sebanyak 30% berijazah S1, tetapi tidak semuanya berlatar belakang studi PAUD.
(5) Namun, belum semua lembaga PAUD di sana memiliki sarana, prasarana, dan anggaran
yang memadai.
Urutan yang tepat agar menjadi teks yang padu adalah … .
A. (3), (1), (4), (2), (5)
B. (3), (2), (5), (4), (1)
C. (3), (5), (2), (1), (4)
D. (1), (3), (4), (2), (5)
E. (1), (5), (2), (3), (4)

33. Bacalah teks berikut!


Efektivitas suatu pemerintahan dalam satu negara ditentukan oleh tiga hal. Pertama, pilihan atas
sistem pemerintahan yang digunakan. Dalam sejarah, kita pernah menerapkan berbagai sistem
pemerintahan, yaitu sistem presidensial, parlementer, sistem presidensial era orde lama dengan
banyak partai, sistem presidensial orde baru dengan 10 partai yang kemudian disederhanakan
menjadi 3 partai, dan sistem presidensial di era reformasi dengan banyak partai sejak Mei 1998
hingga sekarang.Kedua, pilihan atas sistem pemilu. Pemilu dilaksanakan secara langsung, bebas,
dan rahasia. Ketiga, pengaturan waktu penyelenggaraan pemilu. Di Indonesia semua bergantung
pada situasi dan kondisi yang berkembang sejalan dengan teori atau pengalaman positif di
berbagai negara demokrasi. Dalam hal pengaturan waktu pemilu, ada beberapa alasan pemilu
serentak dilaksanakan. Salah satu alasannya adalah meminimalkan terjadinya presiden
minoritas.
Abstraksi (ringkasan) yang sesuai dengan isi teks tersebut adalah ... .
A. Berbagai sistem pemerintahan pernah dilaksanakan di Indonesia yaitu sistem presidensial,
parlementer, presidensial era orde lama, orde baru, dan era reformasi.
B. Banyak alasan pemilu serentak dilaksanakan, yaitu pengalaman positif dan meminimalkan
terjadinya presiden minoritas.
C. Efektivitas suatu pemerintahan ditentukan oleh sistem pemerintahan yang digunakan dan sistem
pemilu yang diterapkan.
D. Pengaturan waktu penyelenggaraan pemilu di Indonesia bergantung pada situasi dan kondisi
yang berkembang sesuai dengan teori atau pengalaman positif di berbagai negara demokrasi.
E. Efektivitas pemerintahan Indonesia ditentukan oleh sistem pemerintahan, sistem pemilu, dan
pengaturan waktu penyelenggaraan pemilu yang disesuaikan dengan situasi dan kondisi.

34. Bacalah teks berikut!


Harga jual komoditas sayuran di Gung Pinto sering mengalami fluktuasi, seperti juga di wilayah
areal pertanian sayuran lainnya di seluruh Indonesia.Saat harga komoditas sayuran turun hingga
tidak menyisakan margin bagi petani, komoditas kopi dan jeruk masyarakat Desa Gung Pinto
berfungsi sebagai tabungan.Hasil penjualan kopi dan jeruk dapat digunakan untuk penutup
kebutuhan rumah tangga dan anak sekolah atau keperluan lainnya. Namun, hanya sedikit petani
yang menanam kopi atau jeruk. Pemerintah sudah membuat program Sinabung Bangkit yang
dibina oleh Pertanian Sehat Indonesia. Namun sistem pengelola dana bantuan tersebut belum
dapat dilaksanakan secara maksimal. Masyarakat sekitar belum terbiasa dengan sebuah program.
Tanggapan logis yang sesuai dengan isi teks tersebut adalah ... .
A. Harga jual komoditas sayuran yang selalu fluktuatif harus disikapi dengan menanam tanaman
lain yang tidak sejenis.
B. Menanam kopi dan jeruk adalah alternatif terbaik agar petani dapat memenuhi kebutuhan hidup
sepanjang tahun.
C. Program Sinabung Bangkit bagi para petani perlu dikaji ulang karena belum dapat dilaksanakan
secara maksimal.
D. Pemerintah harus memaksimalkan pengelolaan dana bantuan dan menyadarkan para petani agar
fluktuasi harga tidak memengaruhi kehidupan mereka.
E. Kesadaran petani perlu ditingkatkan dengan memberikan bantuan dana saat harga komoditas
sayuran menurun.

35. Bacalah teks berikut!


Kemerdekaan Indonesia tak mungkin diraih tanpa kebangkitan literasi lokal yang dimotori Budi
Utomo. Budi Utomo mengembangkan pendidikan dan keaksaraan menjadi pemberdayaan rakyat
pribumi dan pergerakan sosial. Sekolah yang dikembangkan Budi Utomo memodifikasi sekolah
Belanda dengan memasukkan nilai-nilai budaya Jawa. Dengan kemunculan Budi Utomo,
perjuangan melawan penjajah menjadi lebih terorganisasi karena dimotori kaum intelektual
pribumi.

Tanggapan positif yang sesuai dengan isi teks tersebut adalah ... .
A. Indonesia telah merdeka berkat perjuangan para pahlawan dan tidak ada kaitannya dengan
literasi lokal.
B. Kebangkitan literasi lokal dan kemampuan memodifikasi dalam pendidikan oleh Budi Utomo
perlu kita teladani untuk kemajuan bangsa.
C. Memasukkan nilai-nilai budaya Jawa dalam pendidikan adalah hal yang melemahkan
nasionalisme.
D. Kaum intelektual pribumi telah memiliki kemampuan tinggi dan Budi Utomo hanyalah
organisasi lokal.
E. Belanda sangat kuat tetapi kaum intelektual Indonesia lebih kuat sehingga dapat merdeka.

36. Bacalah teks berikut!


Cara membuat:
(1) Kecilkan api. Tuang santan. Masak sambil sesekali diaduk hingga matang.
(2) Sajikan empal gentong bersama pelengkap.
(3) Aduk rata. Tuang kaldu, garam, merica, gula. Masak hingga mendidih.
(4) Rebus daging dalam air sampai empuk. Sisihkan kaldunya 750 ml. Potong-potong
dagingnya.
(5) Panaskan minyak. Tumis bumbu halus, daun salam, dan serai hingga harumlalu
masukkan daging yang telah empuk.
Urutan petunjuk yang tepat adalah ... .
A. (3), (5), (4),(1), (2)
B. (4), (2), (1),(5), (3)
C. (3), (4), (5),(1), (2)
D. (4), (1), (3),(2), (5)
E. (4), (5), (3),(1), (2)
37. Bacalah teks berikut!
Keju merupakan susu yang terkonsentrasi dengan rasa dan tekstur yang beragam. Sama-sama
mengandung kalsium, protein, dan lemak, namun keju juga mengandung lemak jenuh yang
cukup tinggi. Meski terkadang hanya berperan sebagai pelengkap menu, nilai nutrisi dan
kalorinya juga cukup tinggi. Pemenuhan nutrisi dalam tubuh dapat menjaga kebugaran
seseorang.
Kata bermakna proses yang terdapat pada teks tersebut adalah ... .
A. terkonsentrasi
B. berperan
C. pelengkap
D. pemenuhan
E. kebugaran

38. Bacalah teks berikut!


(1) Kelenjar getah bening merupakan organ untuk menjaga kesehatan tubuh dan termasuk dalam
sistem kekebalan tubuh. (2) Di kelenjar tersebut berkumpul sel darah putih untuk menghasilkan
imun yang bisa mencegah infeksi. (3) Organ kelenjar getah bening yang sehat mampu membuat
tubuh tetap sehat.(4) Gangguan kesehatan pada kelenjar getah bening akan menyebabkan
pembengkakan. (5) Pembengkakan pada kelenjar ini dapat diobati sejak dini.

Kalimat yang menyatakan hubungan sebab akibat ditandai dengan nomor ... .
A. (1), (3)
B. (2), (4)
C. (3), (4)
D. (3), (5)
E. (4), (5)

39. Bacalah teks berikut!


Belakangan ini harga bahan-bahan pangan mulai beranjak naik, apalagi menjelang natal dan
tahun baru. Permintaan bahan-bahan pangan melonjak sementara persediaan di pasaran tidak
mampu memenuhi permintaan konsumen. Konsekuensi logisnya ialah harga bahan-bahan
tersebut melonjak naik. Pada titik ini para stakeholder sudah harus mulai memikirkan dan
mengambil tindakan nyata.[ ....]
Kalimat simpulan yang tepat untuk melengkapi teks tersebut adalah ... .
A. Padahal, kondisi seperti ini seharusnya sudah dapat diprediksi oleh kementerian terkait.
B. Akan tetapi, kerja sama kementerian terkait belum mengambil tindakan nyata hingga saat ini.
C. Oleh karena itu seluruh stakeholder harus bertindak lebih cepat agar masalah tidak terjadi
D. Dengan demikian, kita dapat mengatasi masalah kenaikan harga yang secara rutin pasti terjadi.
E. Jadi, untuk menghindari masalah tersebut, sinergi antara kementerian terkait perlu dilakukan.

40. Bacalah teks berikut!


(1) Alif membuka map itu dan membaca isinya.
(2) Konon, Kopral Bambang Budiono ini terbunuh di sebuah penggerebekan 30 tahunlalu.
(3) Alif terperangah dan menaikkan alis mengetahui Mr. Sunyoto nama aslinya adalahKopral
Bambang Budiono.
(4) Di sebuah restoran cepat saji, Alif memegang foto Laras. Dia menatap foto itu lamasekali.
(5) Ditempatkannya foto itu di meja, berdampingan dengan map pemberian Herlam.
Primadonna Angela, 3.
Urutan yang tepat untuk cerita tersebut adalah ... .
A. (2), (3), (1), (5), (4)
B. (4), (5), (1), (3), (2)
C. (4), (2), (1), (3), (5)
D. (5), (3), (1), (2), (4)
E. (5), (1), (3), (4), (2)

41. Bacalah kutipan novel berikut!


“Hanya biola ini milik kami yang masih berharga,”ucap Mak Cik sedih. Nurmi memeluk
biolanya kuat-kuat. Air matanya mengalir. Dia tak rela melepaskan biola itu.
“Nurmi ...,”panggil ibunya.
Nurmi berupaya keras menguat-nguatkan dirinya. Dia mendekati ibuku. Langkahnya terseret-
seret untuk menyerahkan koper biolanya. Air matanya berurai-urai.
“Jangan sekali-kali kau pisahkan Nurmi dari biolanya, Maryamah. Kalau berasmu habis, datang
lagi ke sini.”
Nurmi cepat-cepat menarik tangannya dan kembali memeluk biolanya. Dia tersedu-sedan.
Mata Arai berkaca-kaca melihat Mak Cik bergandengan tangan dengan anak-anaknya sambil
menenteng setengah karung beras.
Andrea Hirata: Sang Pemimpi
Amanat yang terdapat dalam kutipan novel tersebut adalah ... .
A. Jangan berupaya memisahkan benda kesayangan dari seorang anak.
B. Diperlukan pengertian untuk dapat membahagiakan seorang kerabat.
C. Menangislah untuk menunjukkan rasa bahagia karena perlakuan seseorang.
D. Memberikan beras adalah bagian dari sebuah pengertian sosial.
E. Belajar dari kesulitan orang lain agar selalu pandai bersyukur.

42. Bacalah kutipan novel berikut!


Mama tidak terlalu sibuk, tapi dia terlalu banyak bicara. Bukan kepada saya. Tapi pada orang
lain. Mama saya punya sejuta daya tarik. Bukan saja karena tubuhnya yang cantik selalu berbalut
gaun menarik. Tapi juga karena dia pintar membawa diri di luar rumah. Dia punya bisnis event
organizer, terutama bergerak di bidang pameran lukisan. Di rumah kami, ada lemari besar
khusus untuk menyimpan berkas kerja mama yang sangat banyak.
Unsur intrinsik yang terdapat dalam kutipan novel tersebut adalah ... .
A. tema
B. sudut pandang
C. alur
D. latar
E. amanat

43. Bacalah teks berikut!


“Dalam alam bawah sadar Anda, kata malas itu justru makin terpatri, makin kuat. Kata malas
justru semakin menyelimuti otak Anda. Maka kendati beribu-ribu kali mengazamkan ‘saya tidak
malas’, Anda akan tetap malas. Kenapa? Karena memang ada kata ‘malas’ itu dalam pikiran
Anda. Solusinya adalah, hentikan menyebut ‘saya tidak malas’. Ganti menjadi, ‘saya anak rajin’.
Begitu!”
Hendra mengangguk-angguk. Yang lain terkesima. Aku juga.
Anak Anak Langit, Mohd. Amin MS
Nilai moral yang terdapat dalam cerita tersebut adalah ... .
A. Rasa malas selalu ada di hati setiap manusia, terlebih pada sifat Hendra.
B. Tidak semua anak memiliki rasa malas terlebih sifat rajin.
C. Solusi menghentikan rasa malas yang ada di hati dengan cara rajin berpikir.
D. Menanamkan kata rajin dalam benak kita lebih baik daripada tidak malas.
E. Otak kita selalu diselimuti rasa malas yang sulit dihilangkan.

44. Bacalah teks berikut!


jalan berliku-liku, tanah airku
penuh rambu-rambu, indonesiaku

sepanjang jalanan sepanjang tikungan


sepanjang tanjakan sepanjang turunan
rambu-rambu bermunculan
Hamid Jabbar

Majas yang terdapat dalam kutipan puisi tersebut adalah ... .


A. personifikasi
B. litotes
C. paralelisme
D. klimaks
E. metafora

45. Bacalah kutipan drama berikut!


Atma : Diamlah! Diam kataku! Ini bukan urusanmu! Ini urusan hakim! Urusan hukum!
Urusan pengadilan!
Karoman diam.
Atma : Ada yang namanya kejahatan, ada yang menjadi korban perbuatan jahat, ada yang
harus dijatuhi hukuman, semua itu diatur menurut undang-undang.
Karoman masih diam.
Atma : Aku hanya seorang hakim, seorang abdi hukum. Keadilan harus ditegakkan.
Yang terbukti bersalah harus dihukum!
Watak tokoh Atma pada kutipan drama tersebut adalah ... .
A. galak
B. adil
C. baik
D. ramah
E. tegas

46. Bacalah kutipan novel berikut!


Hujan mulai menetes turun ketika Rana sampai rumahnya. Kilat menyambar-nyambar di langit
kelam. Rana merasa tubuhnya demikian berat. Setelah seharian bekerja di depan komputer. Pak
Arya akan berangkat ke luar negeri sehingga Rana harus menyelesaikan seluruh materi
presentasinya. Rumah sunyi, belum ada tanda-tanda berpenghuni. Ia masuk dan melepas lelah di
sofa.
Andrei Aksana: Cinta Penuh Air Mata
Latar tempat yang terdapat dalam kutipan novel tersebut adalah ... .
A. rumah
B. ruang tamu
C. kantor
D. lobi kantor
E. luar negeri
47. Bacalah kutipan drama berikut!
Lembu
Begitulah. Kecuali keadaan di Tegalwurung! Panji Tumbal berhasil ditawan oleh Pangeran
Kembar. Kepalanya dipenggal. Pangeran Bindi menduduki seluruh Kadipaten Tegalwurung dan
menyatakan menentang kedaulatan Maharaja kita, serta menobatkan dirinya sendiri menjadi
Raja. Pengeran kembar mendukungnya.
Reso
Hm! Ini bukan persoalan remeh.
Dara
Ia bukan putra tertua dari almarhum Sri Baginda Raja yang dulu.
Reso
Atas dasar kekuatan! Setiap orang yang merasa dirinya kuat boleh saja menobatkan dirinya
menjadi Raja. Seperti juga Raja yang dulu mendirikan kerajaan ini. Tinggal soalnya apakah ia
akan bisa membuktikan bahwa dirinya benar-benar yang terkuat di seluruh negara. Bisa tidak ia
menundukkan semua tandingan yang ada.
W.S. Rendra
Inti drama tersebut adalah ... .
A. Dukungan Pangeran Kembar untuk Pangeran Bindi setelah menggulingkan kedaulatan
Maharaja yang sah.
B. Dengan kekuatan yang dimiliki, Pangeran Bindi berhasil menguasai Tegalwurung dan
menentang kedaulatan Maharaja yang sedang berkuasa.
C. Pertempuran antara Pangeran Bindi dan Pangeran Kembar untuk memperebutkan kedaulatan
kerajaan Maharaja.
D. Keberhasilan Maharaja dibantu Reso melawan Pangeran Bindi dalam merebut kembali
kerajaannya.
E. Kekuatan Pangeran Bindi yang tidak ada tandingannya di seluruh negara sehingga dapat
merebut kedaulatan yang sah.

48. Bacalah teks berikut!


Sumber : Para Mujahid Cinta
Penulis : Najieb Kailani
Penerbit : Ihwah Publishing House, 2011
Kota : Jakarta
Penggunaan tanda baca yang tepat dalam penulisan daftar pustaka berdasarkan data buku tersebut
adalah ... .
A. Kailani, Najieb, 2011, Para Mujahid Cinta. Jakarta: Ihwah Publishing House.
B. Kailani, Najieb. 2011, Para Mujahid Cinta, Jakarta: Ihwah Publishing House.
C. Kailani, Najieb. 2011. Para Mujahid Cinta. Jakarta; Ihwah Publishing House.
D. Kailani, Najieb. 2011. Para Mujahid Cinta. Jakarta: Ihwah Publishing House.
E. Kailani, Najieb. 2011. Para Mujahid Cinta. Jakarta. Ihwah Publishing House.

49. Bacalah teks berikut!


(1) Uji coba kantong plastik berbayar di toko dan ritel sejak 21 Februari 2016 mendapat
tanggapan positif dari masyarakat. (2) Sebelum memberlakukan kebijakan ini, Direktorat
Pengelolaan Sampah telah melakukan survei, 5-14 Februari 2016. (3) Banyak konsumen
menghindari penggunaan kantong plastik atau membawa sendiri tas belanja. (4) Plastik berbayar
pun dirasakan sangat murah. (5) Terhadap hasil ini, pemerintah belum memiliki tindak lanjut
kebijakan.
Kalimat yang berisi fakta pada teks tersebut ditandai dengan nomor ... .
A. (1), (2)
B. (1), (3)
C. (2), (3)
D. (2), (4)
E. (3), (5)

50. Bacalah teks berikut!


(1) Sistem pemilu Indonesia saat ini adalah proporsional terbuka. (2) Kerugian sistem
proporsional terbuka karena calon legislatif sekadar mengedepankan popularitas. (3) Pada
umumnya, mereka tidak memiliki pemahaman ideologi dan kualitas yang cukup. (4) Keuntungan
sistem tersebut masyarakat diberi kesempatan yang luas untuk memilih calon yang mereka
inginkan. (5) Untuk menyiasati hal tersebut, sistem itu harus dikombinasikan dengan
proporsional tertutup sehingga mampu memperkuat sistem presidensial.
Kalimat yang berisi opini penulis dalam teks tersebut ditandai dengan nomor ....
A. (1)
B. (2)
C. (3)
D. (4)
E. (5)

SOAL USBN BAHASA INDONESIA PAKET 2


KELAS XII

1. Cermatilah paragraf berikut!

(1) Lubang biopori belakangan ini ramai dibicarakan sebagai salah satu bentuk kepedulian kita
terhadap kelestarian lingkungan. (2) Secara alami, biopori adalah lubang-lubang kecil pada tanah
yang terbentuk akibat aktivitas organism dalam tanah, seperti cacing atau pergerakan akar-akar
dalam tanah. (3) Lubang tersebut akan berisi udara dan menjadi jalur mengalirnya air. (4) Jadi, air
hujan tidak langsung masuk ke saluran pembuangan air, tetapi meresap ke dalam tanah melalui
lubang tersebut. (5) Banyaknya pepohonan tidak selalu mengartikan akan ada banyak air di dalam
tanah.

Kalimat utama paragraf tersebut adalah ….

A. (1)
B. (2)
C. (3)
D. (4)
E. (5)

2. Kalimat yang tidak padu dalam paragraf tersebut terdapat pada nomor ….
A. (1)
B. (2)
C. (3)
D. (4)
E. (5)

Cermati paragraf berikut ini!

(1) Keberadaan Saung Angklung Udjo (SAU) meruapakan salah satu upaya melestarikan dan
mengembangkan musik angklung sebagai seni budaya tradisional Sunda. (2) SAU didirikan pada
tahun 1966 ole seniman Sunda, Udjo Ngalagena bersama istrinya, Uum Sumiati. (3) Dalam
perjalanannya, SAU mengilustrasikan alam dan budaya dalam keharmonisan, jati diri kearifan
lokal budaya Sunda. (4) SAU merupakan tempat pertunjukan terpadu yang terdiri atas tempat
pertunjukkan, pusat kerajinan bambu, dan alat instrument bambu. (5) Pemerintah harus
memperhatikan eksistensi dan perkembangan angklung sebagai sebuah warisan budaya nasional.

3. Kalimat fakta dalam paragraf di atas terdapat pada nomor ….


A. (1) dan (2)
B. (2) dan (4)
C. (3) dan (4)
D. (3) dan (5)
E. (4) dan (5)

4. Simpulan yang sesuai dengan isi paragraf tersebut adalah …


A. Musik angklung merupakan satu-satunya seni tradisional Sunda.
B. Musik angklung milik seniman budaya Sunda.
C. Musik angklung menyatukan alam dan budaya Sunda.
D. Musik angklung merupakan salah satu warisan budaya nasional.
E. Musik angklung sebagai seni budaya adat Sunda.

Cermati paragraf berikut ini untuk menjawab soal nomor 5 dan 6!

Sekarang, investasi yang menguntungkan adalah investasi emas. Emas sangat bernilai tinggi
karena tidak dapat dibuat oleh manusia. Selain itu, sediaan emas juga semakin hari semakin
sedikit. Hal ini membuat emas sangat bernilai dan trend-nya cenderung untuk naik. Meskipun
kadang-kadang dapat terjadi gejolak turun, itu hanyalah sesaat dan pada akhirnya akan naik
kembali. Boleh dikatakan, hampir setiap tahun nilai jual emas semakin tinggi.

5. Ide pokok paragraf tersebut adalah ….


A. keuntungan investasi emas
B. tingginya nilai emas
C. sedikitnya persediaan emas
D. kecenderungan kenaikkan harga
E. nilai jual emas
6. Makna kata investasi yang digunakan dalam paragraf tersebut adalah …
A. Penghematan dilakukan untuk menjaga pemborosan.
B. Pengaturan pengeluaran untuk penghematan keluarga.
C. Pemutaran modal untuk memperoleh keuntungan.
D. Penyimpanan barang yang tidak menyebabkan kerugian.
E. Penyimpanan kekayaan agar terhindar dari pemborosan.

Cermatilah kutipan hikayat berikut untuk soal nomor 7 dan 8!

Maka prajurit itu pun segeralah berlari-lari, serta ditikamnya dada Tun Tuah diperturut-turut.
Tetapi Tun Tuah itu orang yang tahu bermain senjata pada barang permainan, maka Tun Tuah
segeralah melompat ke kiri dan ke kanan, menyalahkan tikam prajurit itu. Maka Tun Tuah
melompat ke hadapan prajurit itu serta menghunus keris pendeknya, serta melompat ke kiri dan ke
belakang prajurit itu, lalu ditikamnya dada prajurit itu terus ke belakang, maka Tuan Tuah pun
melompat serta menangkisnya katanya, “Cih! Manatah engkau kata hendak alihkan aku?” Maka
prajurit itu pun rebah tersungkur di hadapan paseban itu. Maka Tun Tuah pun segeralah
menyarungkan kerisnya, lalu naik menyembah Seri Batara dan Raja Malaka.

7. Nilai moral yang terdapat dalam kutipan sastra Melayu Klasik tersebut adalah …
A. Membunuh lawannya untuk menjadi pemenang.
B. Menyembah batara dan raja sebagai penghormatan.
C. Penghinaan terhadap lawannya yang sudah tidak berdaya.
D. Kematian seorang prajurit yang membela negaranya.
E. Seorang prajurit yang tidak sopan karena tidak mau menghormati raja.

8. Karakteristik karya sastra Melayu Klasik dalam kutipan tersebut adalah …


A. Petualangan seorang panglima perang dalam mencari daerah jajahan.
B. Penggunaan bahasa Melayu yang baik sehingga mudah dimengerti.
C. Pemakaian kata penghubung yang tidak tepat pada setiap kalimat.
D. Penggunaan kalimat efektif yang terasa kaku dan monoton.
E. Peperangan yang selalu menyengsarakan rakyat kecil.

9. Cermati penggalan biografi berikut!


Nama Emil Salim dan lingkungan hidup seolah tak bisa dipisahkan. Kepeduliannya pada
lingkungan membuat Emil Salim dipercaya sebagai menteri yang mengurusi lingkungan hidup.
Rupanya kepedulian Emil pada lingkungan karena pengaruh ibundanya. Sang ibunda juga
memberikan filosofi tentang hubungan manusia yang mendalam terhadap mantan Menteri
Pengawasan Pembangunan dan Lingkungan Hidup ini. Besarnya perhatian pada lingkungan,
membuat Emil mendapat penghargaan dari World Wide Fund (WWF) The Leader of Living Planet
Award. Emil Salim mendapatkan penghargaan yang sama dengan yang didapatkan mantan Sekjen
PBB Kofi Annan karena sumbangannya terhadap lingkungan.

Keteladanan Emil Salim pada penggalan biografi tersebut adalah …

A. Mendapatkan kepercayaan sebagai menteri yang mengurusi lingkungan hidup.


B. Kepedulian Emil Salim terhadap lingkungan dilakukan sepanjang hidupnya.
C. Patuh menjalankan filosofi tentang hubungan manusia yang diberikan ibundanya.
D. Mendapatkan penghargaan dari World Wide Fund (WWF) The Leader of Living Planet Award.
E. Mendapatkan penghargaan yang sama dengan apa yang didapatkan mantan Sekjen PBB, Kofi
Annan.

Perhatikan tabel berikut untuk soal nomor 10 dan 11 dengan saksama!

Konsumsi Tepung Terigu Indonesia 2009-2015

Tahun Konsumsi (Metrik Ton) Pertumbuhan (%)

2009 3.800.000 -

2010 3.967.200 4,40

2011 4.383.756 10,50

2012 4.765.879 7,12

2013 5.057.409 7,06

2014 5.431.896 7,00

2015 5.893.607 8,50

10. Pernyataan yang sesuai dengan tabel di atas adalah ...


A. Tahun 2009 konsumsi tepung terigu Indonesia lebih tinggi daripada tahun 2010.
B. Pertumbuhan konsumsi tepung terigu Indonesia tahun 2011 lebih tinggi daripada tahun 2010.
C. Pada tahun 2012-2013 pertumbuhan konsumsi tepung terigu Indonesia menurun tajam.
D. Tahun 2014 pertumbuhan konsumsi tepung terigu Indonesia, lebih besar daripada tahun 2015.
E. Tidak ada peningkatan pertumbuhan konsumsi tepung terigu Indonesia rentang waktu 2014-2015.

11. Simpulan yang tepat berdasarkan tabel tersebut adalah ...


A. Dalam rentang waktu 2009-2015 pertumbuhan konsumsi tepung terigu Indonesia paling rendah
terjadi tahun 2012.
B. Dalam rentang waktu 2009-2015 pertumbuhan konsumsi tepung terigu Indonesia tidak pernah
menurun.
C. Dalam rentang waktu 2009-2015 pertumbuhan konsumsi tepung terigu Indonesia selalu menurun.
D. Konsumsi tepung terigu Indonesia rentang waktu 2009-2015 terus meningkat, tetapi
pertumbuhannya naik turun.
E. Konsumsi tepung terigu Indonesia selalu mengalami naik turun sejak 2009-2015.

Bacalah tajuk rencana berikut untuk soal nomor 12 dan 13 !

Penggunaan jasa penerbangan tentu berharap semua maskapai mencapai standar keselamatan
excellent. Berbeda dengan bus kota yang boleh mogok di tengah jalan, bagi transportasi udara,
kerusakan mesin dan kekacauan sistem pascalepas landas adalah dosa tersebesar. Maskapai
penerbangan tidak boleh terjebak dalam perang tarif. Liberalisasi dalam pasar bebas tidak berarti
kebebasan dalam mematikan pesaing dalam menerapkan tarif serendah-rendahnya. Karena kalau
itu terjadi, dan standar keselamatan dikorbankan, maskapai penerbangan sejatinya tengah
mematikan penggunaan jasa dalam arti harfiah.

12. Opini redaksi yang terdapat dalam penggalan tajuk rencana tersebut adalah ...
A. Maskapai penerbangan harus memiliki standar keselamatan dan tidak boleh terjebak dalam
perang tarif.
B. Transportasi udara yang mengalami kerusakan mesin merupakan bencana dan dosa besar.
C. Maskapai penerbangan tidak boleh melakukan liberalisasi dalam pasar bebas.
D. Jika satandar keselamatan dikorbankan sejatinya telah menerapkan tarif serendah-rendahnya.
E. Selama ini maskapai penerbangan terus-menerus menyeramkan bagi pengguna jasa.
13. Keberpihakan penulis dalam tajuk tersebut adalah ….
A. maskapai penerbangan
B. pengusaha penerbangan
C. pengguna jasa penerbangan
D. pemerintah
E. pengelola penerbangan

14. Bacalah paragraf berikut!


Gara – gara memakai onderdil “bersertifikasi palsu”, Pemerintah Korea Selatan terpaksa
menghentikan operasi dua pembangkit listrik tenaga nuklir. Akibatnya, negara terancam
kekurangan pasokan listrik memasuki musim dingin
Kalimat tanya yang sesuai dengan isi teks tersebut adalah . . .
A. Mengapa pembangkit listrik tenaga nuklir di Korea Selatan menggunakan onderdil bersertifikat
palsu?
B. Apa saja yang menyebabkan negara terancam pemadaman listrik?
C. Bagaimana sikap Pemerintah Korea Selatan atas penggunaan onderdil palsu pada pembangkit
listrik tenaga nuklir?
D. Berapa banyak kerugian yang diderita Pemerintah Korea Selatan akibat penghentian operasi dua
pembangkit listrik tenaga nuklir?
E. Siapa yang mengetahui pertama kali bahwa dua pembangkit listrik tenaga nuklir menggunakan
onderdil palsu?

15. Bacalah penggalan gurindam berikut dengan saksama!

Keaiban orang jangan dibuka,

Keaiban diri hendaklah sangka

Maksud gurindam tersebut adalah …

A. Tidak baik menceritakan keburukan orang, lebih baik intropeksi diri.


B. Janganlah membuka aib orang lain, tetapi bantulah menutupinya.
C. Aib orang jangan dibuka, sementara aib sendiri semakin menjadi
D. Aib orang diceritakan, tidak disadari terbuka pula aib sendiri.
E. Menceritakan keburukan orang, sama halnya menceritakan keburukan sendiri.

16. Bacalah puisi berikut dengan cermat!

Mimbar

Dari mimbar ini telah dibicarakan

Pikiran-pikran dunia

Suara-suara kebebasan

Tanpa ketakutan

Dari mimbar ini diputar lagi

Sejarah kemanusiaan

Pengembangan teknologi

Tanpa ketakutan

Di kampus ini

Telah dipahatkan

Kemerdekaan

Segala despot dan tirani

Tidak bisa merobohkan

Mimbar kami

(Taufik Ismail)

Suasana dalam puisi tersebut adalah ….

A. ketakutan
B. khawatir
C. keberanian
D. kekejaman
E. kebebasan

17. Bacalah kutipan cerpen berikut!


Tiap hari Cenana membantu ibunya memarut puluhan butir kelapa. Mereka membuat minyak
kelapa yang pagi-pagi sekali harus disetor ke pasar. Pulang agak siang, Cenana langsung
membantu ibunya memasak. Untuk mencari tambahan nafkah satu-dua ribu perak, Jero Sandat
membuka warung kecil yang menjual lawar dan pepes tlengis. Keringat kedua perempuan itu
benar-benar diperas habis untuk sekadar bisa hidup. Bagi Cenana, sekolah hanya impinan yang
terlalu mewah.

(Perempuan-Perempuan Matahari, Oka Rusmini)

Pendeskripsian watak tokoh Cenana seorang yang rajin pada kutipan novel tersebut adalah ….

A. gambaran fisik tokoh


B. ucapan tokoh lain
C. tindakan tokoh
D. pikiran tokoh
E. dialog antartokoh

18. Bacalah puisi berikut!


Sajak

O, bukannya dalam kata yang rancak,

Kata yang pelik kebgusan sajak,

O, pujangga, buang segala kata,

Yang ‘kan Cuma mempermainkan mata,

Dan hanya dibaca sepintas lalu,

Karena tak keluar dari sukma,

Seperti matahari mencintai bumi,

Memberi sinar Selama-lamanya,

Tidak meminta sesuatu kembali,

Harus cintamu senantiasa

Sanusi Pane

Makna kata lambang matahari adalah ….

A. kehidupan
B. kenyamanan
C. kesombongan
D. keperkasaan
E. kekuatan
Cermatilah kutipan cerpen berikut untuk soal nomor 19 dan 20!

Ketika ia berjalan di koridor meneruskan perjalanan menuju ruang UKS untuk meminta maaf, saat
sampai di depan UKS ia melihat Bu Sartika sedang menasehati dengan penuh kasih sayang.
Menyaksikan hal tersebut Salman merasa iri sebab selama ini Bu Guru Sartika tidak pernah lagi
menyediakan waktu untuknya. Pada saat ia menuju ruangan tersebut, ia berkata, “Oh, ini yang Ibu
lakukan sehingga tidak ada waktu untuk mengajari Salman lagi,” ucapnya dengan nada tinggi.

“Bukan begitu Man, Ibu melakukan ini karena kasihan melihat Anjas seperti ini,” tegasnya.

“Alah, jangan berbohong deh, Bu.” bantahnya sembari berjalan.

Ketika keluar meninggalkan ruangan tersebut, ia menangis sampai akhirnya ia tertunduk di sebuah
bukit kecil di belakang sekolah.

19. Keterkaitan peristiwa dalam kutipan cerpen tersebut dengan kehidupan sehari-hari adalah …
A. Perilaku meminta maaf atas segala kesalahan.
B. Seseorang yang membeda-bedakan kasih sayang terhadap anak.
C. Perasaan cemburu pada seseorang.
D. Kekecewaan seseorang karena kegagalan.
E. Penyesalan diri karena telah menyakiti orang lain.

20. Bagaimana sikap Bu Sartika melihat sikap dan tindakan Salman?


A. Marah-marah dan tidak mau menerima tuduhan Salman
B. Acuh tak acuh melihat perlakukan Salman kepadanya.
C. Sangat tersinggung karena Salman tidak sopan kepadanya.
D. Sakit hati karena Salman bertindak kasar.
E. Tetap bersabar dan menjelaskan alasannya kepada Salman

21. Bacalah kutipan novel berikut!

(1) Zamzani sendiri pernah melihat kamus yang hebat di pedagang buku bekas kaki lima di
Tanjong Pandan. (2) Kamus itu adalah Kamus Bahasa Inggris Satu Miliar Kata.(3) Sejak melihat
kamus itu dan mengenang keinginan putrinya, membeli kamus telah menjadi impian Zamzani dari
hari ke hari. (4) Ia bekerja lebih keras di ladang tambang dan menambah penghasilan dengan
berjualan air nira setiap ada pertunjukkan orkes Melayu. (5) Hari Sabtu ia ke laut mencari karang
untuk dijual di pasar ikan. (6) Hari Minggu ia berjualan tebu yang ditusuk dengan lidi. (7) Setelah
berbulan-bulan seperti itu dan memfokuskan pikirannya hanya untuk membeli kamus bahasa
Inggris, akhirnya Zamzani punya uang lebih. (8) Dengan ceria ia sampaikan pada putrinya.

Latar suasana gembira dalam kutipan tersebut dibuktikan dengan kalimat …

A. (1) dan (2)


B. (2) dan (3)
C. (4) dan (5)
D. (6) dan (7)
E. (7) dan (8)
22. Perhatikan penggalan teks naskah drama berikut!

Maskun : Anak itu kian hari menjadi liar! Kau mesti peringatkan Suhita!

Mardillah : Ada apa dengan Suhita, Pak? Tadi pun dia mengeluh karena kau marah lagi. Sudah
selayaknya kau berdamai dengannya.

Maskun : Dia yang harus berdamai dengan aku. (Terdiam sejenak). Anak itu seperti bukan
anakku …

Mardillah : (Mardillah duduk tertunduk) Mengapa Kau berperasaan begitu?

Maskun : (Berdiri menghela napas) Tak tahu aku. Mulut anak itu semakin berbau racun. Barulah
dia berkata, rumah ini rumah penjara. Dan akulah kepala penjaranya. (pandangan
mata Maskun mendakwakan tuduhan kepada isterinya. Mardillah terkejut takut
menerima tatapan mata suaminya)

Amanat yang terdapat dalam kutipan penggalan teks drama di atas adalah ….
A. janganlah membantah kata orang tua
B. seorang anak harus patuh terhadap orang tua
C. seorang anak harus bersikap tegas terhadap orang tua
D. hendaknya orang tua menuruti kehendak anaknya
E. orang tua seharusnya menjadi panutan bagi anak-anaknya

23. Bacalah penggalan novel berikut!


Mami yang muncul bersama Tini dan kedua adik tirinya sudah sampai ke halaman rumah bu
MantrI.Ia berjalan menunduk.Marni segera menjadi pusat perhatian.Semua orang yang berada
disana diam.Hening.Ian suasana tiba-tiba berubah mencekam seolah-olah mereka menunggu
sesuatu yang luar biasa akan terjadi.
Dipintu Marni mengucapkan salam.Pelan sekali.Kemudian matanya berkeliling mencari laki-laki
yang baru pulang dari pengasingan di Pulau B itu.Karman bangkit dari duduknya.Gerakannya
tenang saja,tetapi nyata sekali tangannya bergetar.Perempuan yang selama 12 tahun dirindukan
sekarang berada di hadapannya.Yang selama itu pula menjadi angan-angannya. Yang tanpa dia
Karman hampir bertekad memusnahkan dirinya sendiri. Dimata Karman,Marni tetap cantik,atau
bahkan lebih cantik karena kini sudah matang.
Apa yang sedang menyapu perasaan Karman demikian pula yang dirasakan oleh Marni.Dan
apabila Karman berhasil menguasai perasaannya,Marni tidak.Bagaimana juga ia seorang
perempuan.Ada sesuatu yang terasa mendidih dan meluap dalam dada Marni.Tubuhnya
bergoyang.Lalu ia bergerak kea rah Karman.Mulutnya terbuka.Tetapi ada kekuatan yang
mencegahnya bergerak lebih lanjut.dari mulut Marni terdengar suara tertahan,”mas. . .Mmmmas
Karman!”
Hanya itu.karena kemudian Marni tidak lagi bergerak.Ia berhenti dalam keadaan yang
ganjil.Tangannya seakan-akan hendak menggapai kedepan,tetapi tubuhnya condong
kebelakang.Beberapa detik Marni tetap demikian.Lama-lama tubuhnya goyang.Karman cepat
menangkap tubuh marni sebelum perempuan itu roboh ketanah.
Sumber: Kubah,Ahmad Tohari

Konflik yang terdapat dalam kutipan novel tersebut adalah …


A. Marni menjadi pusat perhatian di kalangan orang-orang yang berada di rumah itu.
B. Hati Marni bergejolak ketika ia mengetahui Karman, bekas suaminya , telah kembali dari
pengasingan.
C. Marni harus bertemu dengan Bu Mantri,orang yang berpengaruh di kampung itu.
D. Marni meninggalkan suaminya yang sedang sakit di rumah.
E. Marni harus berbohong kepada bekas suaminya bahwa ia sudah menikah kembali.

24. Cermatilah kutipan cerpen berikut!


Hal itu terjadi seminggu yang lalu. Pak Hardi seperti biasa mengendarai sepeda ontelnya
berkeliling mencari dagangan pisang. Ia menerobos kebun-kebun dan ladang-ladang petani.
Karena kepergiaanya dengan pikiran yang kalut, ia tidak pikir panjang menebang pisang milik
orang lain yang belum dikenalnya. Dua tandan pisang nagka dibawanya ke bos pisang. Tidak sadar
kalau ada orang yang mengetahui perbuatannya. Suatu hari dari kejadian itu, Pak Hardi dijemput
paksa oelh seorang polisi yang maish sangat muda. Bahasanya cukup tegas. Hanya beberapa kata
yang diucapkannya. Pak Hardi pun tak berkutik. Tanpa perlawanan ia mengikuti sang polisi. Ia
sadar kalau ia melanggar hokum karena telah mencuri pisang milik orang lain.

(Dua Sisi Mata Pisau, Lanang A.)

Nilai positif sesuai dengan karakter tokoh Pak Hardi adalah …

A. Menyadari kesalahannya karena telah mengambil milik orang lain.


B. Menurut kata kepala polisi karena dia takut melawan.
C. Mencari kehidupan dengan menerobos kebun-kebun orang.
D. Menebang pisang orang sebelum meminta kepada pemiliknya.
E. Tidak membalas perlakuan polisi yang telah menangkapnya.

25. Bacalah paragraf narasi berikut!


Aku menutup novel yang telah habis kubaca. Mataku mulai terasa perih setelah semalaman
terpaku pada baris demi baris yang kubaca. Kuregangkan tubuhku sejenak, lalu aku berdiri dan
berjalan ke bordes gerbong kereta. Kulemparkan pandanganku ke luar kereta. Perlahan, langit
hitam mulai memudar … Udara di luar kereta terasa sejuk. Bau tanah pagi khas di antara sawah
dan hujan jati. Ah, rindu pada desaku semakin membongkah.

Kalimat yang tepat untuk melengkapi paragraf tersebut adalah …

A. Di langit timur bulan purnama tersenyum menyambut malam.


B. Langit subuh mulai terlihat di antara rimbun pepohonan pohon jati.
C. Jalan desa masih lengang dan gemericik air terdengar jelas.
D. Anak-anak sekolah terlihat berlarian riang di jalan desa.
E. Pak tani menggiring lembunya ke sawah sambil membawa bajak.

26. Cermatilah paragraf berikut!


Pertumbuhan konsumen selular di negeri ini meningkat sangat cepat. Tidak adalagi mahasiswa
yang tidak memiliki telepon selular. Begitu juga dengan siswa SMA, SMP bahkan SD telah
menggunakan telepon selular untuk berkomunikasi dengan orang tua mereka. Pedagang keliling,
seperti tukang bakso, tukang cendol menggunakan telepon selular sebagai sarana penjualaan. Di
pelosok pedesaan para orang tua juga sudah menggunakan telepon selular untuk berkomunikasi
dengan anak mereka. Yang bekerja sebagai tenaga kerja luar negeri, seperti di Hongkong,
Thailand, arab saudi dan Singapura. ....

Kalimat simpulan yang tepat untuk melengkapi paragraf tersebut adalah ...
A. Semua lapisan masyarakat menggunakan telepon selular sebagai alat komunikasi.
B. Pengguna telepon selular tidak asing bagi masyarakat saat ini khususnya indonesia.
C. Telepon selular sebagai alat komunikasi jarak jauh sudah digunakan di mana – mana.
D. Berkomunikasi dengan telepon selular lebih mudah dan murah biayanya.
E. Telepon selular lebih praktis dan mudah dibawa ke mana – mana oleh siapa saja.

27. Cermatilah paragraf berikut!

Berkunjung ke kebun binatang Taman Safari, selain mengamati gerak-gerik yang … dari aneka
satwa, wisatawan juga dapat menikmati udara yang … dengan pepohonan rindang di sekitar
taman. Wisatawan juga dapat menikmati keceriaan bersama keluarga, bersantai seraya menyambut
sapaan angin pegunungan. Menikmati suguhan alam yang … merupakan langkah tepat membuat
hidup menjadi lebih hidup.

Frasa yang tepat untuk melengkapi paragraf tersebut adalah ….

A. sangat unik, cukup sejuk, sangat indah


B. cukup unik, sangat indah, paling baik
C. lucu sekali, sangat dingin, amat bagus
D. paling bagus, agak baik, bagus sekali.
E. aneh sekali, paling dingin, sangat baik

28. Bacalah paragraf berikut dengan cermat!

Dia sudah meminta maaf …, tetapi permintaan maafnya tetap tidak dihiraukan. Dia pun marah dan
… pintu sehingga suara gaduh terdengar dari kamar. Anak terkecilnya… histeris karena ketakutan.

Kata ulang yang tepat untuk melengkapi paragraf tersebut adalah ….

A. sekali-kali, memukul-memukul, jerit-jerit


B. berkali-kali, memukul-mukul, menjerit-jerit
C. berkali-kali, mukul-memukul, menjerit-jerit
D. sekali-kali, memukul-mukul, menjerit-jerit
E. berkali-kali, memukul-mukul, jerit-jeritan
29. Cermatilah paragraf berikut!

Drama mempunyai tiga dimensi, yaitu sastra, gerakan, … ujaran. Oleh sebab itu, drama tidak
disusun khusus untuk dibaca seperti novel atau cerita pendek, …. dibaca untuk diekspresikan …
bentuk mimik dan gerak.

Kata penghubung yang tepat untuk melengkapi paragraf tersebut adalah ….

A. dan, tetapi, dalam


B. serta, namun, mengurai
C. atau, melainkan, tentang
D. atau, tetapi, oleh
E. dan, bahkan, melalui

30. Cermati paragraf berikut!

Peneliti Dr. Cathy Cutler menemukan obat pembasmi sel kanker meskipun memiliki dosis rendah.
Beliau menemukan senyawa epigallocatechin gallate (EGCg) dalam the hijau. Senyawa tersebut
merupakan … yang mampu melawan kanker. Menurut Dr. Cathy, kanker prostat yang … bisa
dengan … dihancurkan oleh senyawa tersebut.

Kata baku yang tepat untuk melengkapi paragraf tersebut adalah ....
A. antioksidasi, aggressive, efektif
B. anti oksidan, agresif, efektif
C. antioksidan, agresif, efektif
D. antitoksin, agresiv, evektif
E. antioksidan, agresif, efektive

31. Cermati paragraf berikut!

Taman Toyota terletak di Jalan Perintis Kemerdekaan, Jakarta Utara. Di taman Toyota banyak
rumput kering dan tumbuh tidak merata. Barangkali banyak orang yang juga tidak sadar akan
keberadaan taman ini. Karena itu, kegersangan taman tidak terlalu menyedot perhatian. Barangkali
persoalan akan berbeda bila taman gersang ini mudah terlihat seperti di Jalan M.H. Tamrin.

Perbaikan kalimat tidak padu (dicetak miring) yang tepat adalah …


A. Letaknnya tertutup jaringan jalan.
B. Taman itu dipenuhi bunga.
C. Di taman ini terdapat pepohonan yang rindang.
D. Taman itu ramai di sore hari karena banyak pengunjung.
E. Letaknya strategis dan terjangkau.

32. Cermatilah kalimat berikut ini!

(1) Program ini tidak terlalu cocok bagi semua siswa.


(2) Program akselerasi terkesan ekslusif karena hanya menerima siswa tertentu.
(3) Siswa menginginkan belajar cepat bisa memanfaatkannya.
(4) Memang, kegiatan belajar perlu disadari oleh kesiapan dan kemampuan diri siswa yang
bersangkutan.
(5) Apabila siswa tidak siap dan tidak mampu mengikuti irama belajar yang serba cepat mereka
tidak dapat mencapai prestasi maksimal.

Urutan kalimat tersebut agar menjadi paragraf eksposisi yang padu adalah …
A. (1), (2), (3), (4), dan (5)
B. (1), (3), (2), (4), dan (5)
C. (2), (1), (3), (5), dan (4)
D. (2), (3), (1), (4), dan (5)
E. (2), (3), (4), (5), dan (1)

33. Cermatilah penutup surat lamaran berikut dengan seksama!

Demikian surat lamaran pekerjaan ini saya buat. Mohon maklum dan terima kasih banyak.

Perbaikan kalimat penutup surat lamaran pekerjaan tersebut adalah ...


A. Atas perhatiannya, saya mengucapkan terima kasih.
B. Atas perhatian Bapak/Ibu, saya mengucapkan terima kasih.
C. Kami ucapkan terima kasih atas perhatian Bapak/Ibu.
D. Atas perhatian Bapak/Ibu, saya menghaturkan terima kasih.
E. Atas perhatian Bapak/Ibu, kami ucapkan banyak terima kasih.

34. Cermatilah iklan berikut!

DIBUTUHKAN SEGERA

Perusahaan industry komponen otomotif roda 2 dan 4 yang berkembang pesat membutuhkan
tenaga kerja, sebagai berikut:

a. Pria max. 27 tahun


b. Min SMU/sederajat
c. Memiliki SIM C.
d. Menyukai tugas lapangan dan pengalaman bid. Customer Goods/Med. Rep/Sparepart min, 1
thn.
Ruko Cempaka Mas blok Q-5
Jalan Letjend Suprapto Jakarta Pusat

Paling lambat 10 hari setelah iklan ini dimuat.

Kompas, 10 September
2017

Kalimat pembuka surat lamaran pekerjaan yang sesuai dengan iklan tersebut adalah …

A. Dengan surat ini, saya melamar pekerjaan di iklan yang dimuat di harian Kompas, 10 September
2017. Dengan ini saya.
B. Berdasarkan penjelasan iklan di harian Kompas, 10 September 2017, saya mengajukan lamaran
terhadap Saudara.
C. Sehubungan dengan iklan yang dimuat pada harian Kompas, 10 September 2017, dengan ini saya
mengajukan lamaran kerja pada perusahaan Saudara.
D. Sesuai dengan iklan, 10 September 2017 tersebut, saya ingin sekali bekerja di perusahaan ini.
Maka saya menulis surat lamaran pekerjaan.
E. Iklan di harian Kompas, 10 September 2017 telah menarik keinginan saya untuk bekerja di
perusahaan Saudara oleh sebab itu saya mengajukan lamaran kerja.

35. Cermati ilustrasi berikut!

OSIS SMA Negeri Global Jaya akan menyelenggarakan pelatihan majalah dinding (mading) di
aula sekolah tersebut. Acara ini akan diselenggarakan pada hari Sabtu, 3 Oktober 2017, pukul
08.00 – 13.00 WIB.

Penulisan pemerian dalam surat undangan yang tepat berdasarkan ilustrasi tersebut adalah ...
A. Hari, tanggal : Sabtu, 3 Oktober 2017
Pukul : 08.00 – 13.00 WIB
Tempat : Aula SMA Negeri Global Jaya
Acara : Pelatihan Majalah Dinding

B. hari/tanggal : Sabtu, 3 Oktober 2017


pukul : 08.00 s/d 13.00 WIB
tempat : Aula SMA Negeri Global Jaya
acara : pelatihan majalah dinding

C. hari, tanggal : Sabtu, 3 Oktober 2017


pukul : 08.00 s.d. 13.00 WIB
tempat : Aula SMA Negeri Global Jaya
acara : Pelatihan Majalah Dinding

D. Hari, tanggal : sabtu, 3 Oktober 2017


Pukul : 08.00 – 13.00 WIB
tempat : Aula SMA NEGERI GLOBAL JAYA
Acara : Pelatihan Majalah Dinding
E. Hari/ tanggal : Sabtu, 3 Oktober 2017
pukul : 08.00 – 13.00 WIB
tempat : Aula SMA Negeri Global Jaya
acara : Pelatihan Majalah Dinding

36. Cermatilah paragraf berikut!

Seorang kepala keluarga sedang memerlukann uang untuk biaya sekolah anaknya. Sebagai
pedagang nasi goring, ia sudah bekerja … untuk menafkahi keluarganya. Jika ada waktu luang,
kadang-kadang ia rela menjadi tukang ojek. Namun, tabungannya ternyata tidak cukup untuk
membiayai anaknya bersekolah sehingga motor satu-satunya yang ia miliki pun harus dijual.

Ungkapan yang tepat untuk mengisi rumpang paragraf di atas adalah ….

A. ringan tangan
B. berat hati
C. makan hati
D. tulang punggung
E. banting tulang

37. Bacalah dialog berikut dengan saksama!

Dua bulan yang lalu buruh nelayan itu tidak melaut karena angin laut sedang pasang. Kini mereka
harus antre medapatkan solar bersubsidi yang pembeliannya dua puluh ton per kapal. Ratusan
kapal penangkap ikan antre untuk membeli solar agar bisa melaut. Ternyata kapal-kapal ittu tetap
tidak dapat melaut karena gagal mendapatkan solar.

Peribahasa yang tepat sesuai isi paragraf tersebut adalah …

A. Bagai pinang di belah dua.


B. Tak ada gading yang tak retak.
C. Sudah jatuh tertimpa tanggal pula.
D. Ada sampan hendak berenang.
E. Air beriak tanda tak dalam.

38. Bacalah paragraf berikut dengan saksama!

Buku “Pengalaman menulis buku nonfiksi”, karya Wisnu Barata berisi bagaimana menyusun
naskah buku dari awal hingga proses penerbitan. Buku ini perlu dimiliki oleh penulis yang ingin
menerbitkan karyanya. Bahasanya enak dibaca dan mudah dipahami.

Kalimat resensi yang tepat berdasarkan ilustrasi tersebut adalah …

A. Buku ini membahas pengalaman penulis berhubungan dengan penerbit dalam menulis buku dari
awal hingga akhir.
B. Ingin buku Anda diterbitkan? Bacalah buku ini, sederhana, singkat, dan banyak contoh yang
dipaparkan.
C. Buku ini terasa istimewa karena dengan bahasa yang mudah dipahami penulis mengulas
pengalamannya dalam menerbitkan buku.
D. Buku ini perlu diteliti orang banyak karena bahasa yang mudah dipahami, terlebih bagi penulis.
E. Siapa yang ingin menerbitkan buku perlu membaca buku ini karena berisi cara menulis buku.

39. Cermatilah paragraf berikut!

Kehidupan modern tampaknya lebih mengedepankan individu daripada gotong royong. Orang-orang
kapital jangan sampai mendobrak kehidupan kita. Oleh karena itu, kita harus melakukan formasi secara
besar-besaran untuk membangun kembali budaya kita yang terkenal dengan gotong royongnya.

Istilah yang tepat untuk mengganti kata yang bercetak miring tersebut adalah ….

A. individualis, kapitalis, reformis


B. individualis, kapitalisme, reformasi
C. indualisme, kapitalisme, reformis
D. individualistis, kapitalisasi, reformasi
E. individualis, kapitalis, reformasi

40. Bacalah paragraf berikut!


Leni berkali-kali ditanya orang tuanya tentang pernikahan. Untuk menghilangkan tekanan dari
orang tuanya, Leni akhirnya berterus terang bahwa ia ingin kuliah lagi mencapaikan gelar doctor.
Akan tetapi, orang tuanya mempercayai pengakuan Leni sebagai upaya Leni menolak lamaran
Robi.

Perbaikan kata berimbuhan yang tercetak miring pada paragraf tersebut adalah ….

A. menghapus, mengejar, menganggap


B. mengusir, menggapai, memandang
C. melepaskan, mendapatkan, memahami
D. menghindari, meraih, menafsirkan
E. mengelabui, memperoleh, menangkap

41. Cermati paragraf berikut!

Di sudut kanan lapangan sekolah itu tampak siswa duduk berkelompok. Setiap kelompok asyik
melakukan … rutin setiap Sabtu sesuai kegiatan ekstrakurikuler yang dipilihnya. … sekali mereka
mengikutinya. Banyak guru yang menilai kegiatan ini sangat … untuk mengisi waktu luang
mereka.

Kata serapan yang tepat untuk melengkapi paragraf tersebut adalah ….

A. aktivitas, antusias, efektif


B. aktif, antusiasme, efektivitas
C. aktivitas, antusiasi, efektifitas
D. keaktifan, keantusiasan, keefektifitasan
E. aktif, antusias, efektif

42. Perhatikan dialog berikut!

Wayan : “Sabar Gusti, kenapa Gusti gelap mata? Gusti telah menghantam semua

orang dengan utang. Satu miliar dan … (menoleh ke belakang dan heran) di

mana Nyoman, Gusti?”

Gusti Biang :”Dia sudah pergi, pagi buta. Dia tidak akan mengganggu kita lagi.”

Wayan : ….

Gusti : “Ya, kita sudah terlepas dari bahaya karena ia sudah pergi.”

Wayan : “Terlepas? Justru bahaya itu sekaranglah baru mulai, Gusti.”

Kalimat yang tepat untuk melengkapi bagian rumpang dialog tersebut adalah …

A. Maksud Gusti, dia sudah pergi dan tiyang tidak melihatnya?


B. Saya tidak yakin kalau dia sudah pergi dari sini, Gusti.
C. Tidak mungkin dia sudah pergi, Gusti.
D. Bagaimana dia bisa pergi dari sini?
E. Apa Gusti tidak melarangnya?

43. Cermatilah pantun berikut!

Kue bolu kue donat

Dimakan saat hujan kilat

Walaupun capek dan penat

…..

Larik yang tepat untuk melengkapi pantun tersebut adalah ….


A. Kita harus taat adat
B. Kita harus tetap giat
C. Jangan pernah tinggalkan salat
D. Jangan suka bermain gulat
E. Senang hati melihat gulat

44. Cermatilah topik karya ilmiah berikut!

Topik: Pengaruh Kegiatan Ekstrakurikuler terhadap Pembentukan Kepribadian Siswa

Kalimat rumusan masalah yang tepat sesuai dengan topik tersebut adalah …

A. Adakah pengaruh kegiatan ekstrakurikuler terhadap pembentukan kepribadiaan siswa?


B. Mengapa kegiatan ekstrakurikuler berpengrauh terhadap pembentukan kepribadiaan siswa?
C. Siapa yang dapat membentuk kepribadiaan siswa?
D. Bagaimana pengaruh kegiatan ekstrakurikuler terhadap pembentukan kepribadiaan siswa?
E. Bagaimana peran siswa dalam membentuk kepribadiaan pada kegiatan ekstrakurikuler?

45. Cermatilah topik karya ilimiah berikut!

Topik karya tulis: pencegahan narkoba di kalangan remaja

Kalimat latar belakang yang sesuai dengan topik tersebut adalah …

A. Orang tua prihatin atas pergaulan bebas para remaja


B. Orang tua dan guru seharusnya dapat mencegah penyimpangan remaja.
C. Pengaruh media elektronik dan internet bagi remaja.
D. Semakin banyak remaja terlibat dalam penyalahgunaan narkoba.
E. Kurangnya penyuluhan tentang narkoba di kalangan remaja.

46. Bacalah kutipan naskah pidato berikut!

Hadirin yang berbahagia,

Bencana alam terjadi di mana-mana sehingga sangat merugikan bagi manusia dan alam sekitarnya.
Kita kehilangan harta benda ataupun orang-orang yang kita sayangi. Mengapa bencana alam
terjadi? Apakah karena ulah manusia sendiri yang mulai kurang peduli dengan lingkungan? Hutan
digunduli tanpa perhitungan, pohon-pohon dijarah oleh manusia serakah, sampah dibuang di
saluran-saluran air. Pembangunan dilakukan tanpa memperhatikan lingkungan. Kita tidak ingin
bencana alam terjadi lagi …
Kalimat ajakan yang tepat untuk melengkapi teks pidato tersebut adalah …

A. Marilah kita jaga lingkungan kita dengan bersikap lebih bijak dan peduli terhadap alam sekitar
kita.
B. Jangan mengulang sikap-sikap yang tidak terpuji karena kita tidak ingin bencana terulang lagi.
C. Sebaiknya bersatu padu menjaga dan melestarikan kekayaan bangsa dan tanah air kita.
D. Lingkungan harus dijaga dengan baik sehingga akan memberikan manfaat kepada kita.
E. Manusia dapat bersikap bijaksana dalam mengolala lingkubgan dan berpartisipasi terhadap
lingkungan sekitar kita.

47. Bacalah dengan cermat puisi berikut!

Di Jari Manismu Ada Rindu

Kurangkai kata agar jadi sajak berkalung

Kurangkai bait-baitnya agar hati menyatu

Yang menulis cincin tunangan

Di jari manismu ada rindu

Hammami Adaby

Kalimat kritik sastra yang sesuai dengan puisi tersebut adalah …

A. Hubungan makna antara larik ketiga ke larik keempat ada lompatan.


B. Kata-kata puitis yang melambung selalu mewarnai perasaan dalam khayalan penyair.
C. Pilihan kata penyair terlalu tinggi sehingga sulit dimaknai secara lugas.
D. Makna kata tidak tampak menggambarkan perasaan penyair, terkesan sangat biasa.
E. Larik-larik bermajas tidak memberikan nuansa keindahan yang menawan hati.

48. Bacalah topik karya tulis berikut!

Judul karya tulis : Upaya meningkatkan minat baca dan kreativitas siswa kelas X SMA jaya
tahun

2017

Perbaikan penulisan judul karya ilmiah tersebut yang tepat adalah ….

A. Upaya Meningkatkan Minat Baca Dan Kreativitas Siswa Kelas X SMA Jaya Tahun 2017
B. Upaya Meningkatkan Minat Baca dan Kreativitas Siswa Kelas X SMA Jaya Tahun 2017
C. Upaya Meningkatkan Minat Baca dan Kreativitas Siswa kelas X SMA Jaya Tahun 2017
D. Upaya Meningkatkan Minat Baca dan Kreativitas Siswa kelas X SMA Jaya tahun 2017
E. Upaya Meningkatkan Minat Baca Dan Kreativitas Siswa kelas X SMA Jaya Tahun 2017

49. Bacalah penggalan puisi berikut!

Ketika laut menderu

dan detak …

ada kerinduan di kalbu

ada harapan ingin bertemu

(K. Muharlin)

Kalimat yang tepat untuk melengkapi bagian rumpang puisi tersebut adalah ….

A. angin menyerbu
B. lonceng menyeru
C. jantung menggebu
D. suara menganggu
E. kata melagu

50. Cermati paragraf esai berikut!

Midah terpikat lagu keroncong ketika ia bermain di jalanan. Ia langsung jatuh cinta pada
keroncong, tapi ayahnya menentangnya. “Musik haram!” kata ayahnya. Tak lama berselang, sang
gadis manis dinikahkan dengan seorang haji. Haji Terbus, orang terkenal di Cibatok. Pernikahan
itu tidak lama. Maidah melarikan diri.
(Midah Sang Gadis Bergigi Emas, Pramoednya
A. T)

Kalimat esai yang sesuai dengan penggalan novel tersebut adalah …


A. Midah dan sosoknya adalah tokoh yang tertutup yang aneh kalau diterjemahkan sebagai tokoh
kontroversial seperti tokoh-tokoh dalam novel Pram.
B. Midah menulis dari sejarah untuk sejarah, dia menulis berupa dasar dan hasil pengamatan yang
kuat melalui proses perenungan.
C. Kebangkitan sejarah dan perjuangan rakyat Indonesia selama dan sesudah era kolonialisme
Belanda menjadi hiasan dalam novel-novelnya.
D. Novel-novelnya tidak menyatakan sikap tegas atas ketidakadilan dan penindasan.
E. Untuk memahami pembentukan jati diri, tampak Pram memberikan pelajaran bahwa keterpaksaan
dapat merugikan diri.

SOAL USBN BAHASA INDONESIA PAKET 3


KELAS XII
1. Bacalah paragraf berikut dengan saksama!
Sebanyak 195 kabupaten di Indonesia pernah menjadi daerah endemis penyakit kaki
gajah. Penyakit kaki gajah adalah infeksi bersifat menahun, disebabkan oleh cacing
filaria yang hidup dalam tubuh manusia. Cacing filaria terdapat di saluran getah bening
terutama di pangkal paha, ketiak, dan saluran getah bening lainnya.

Makna istilah endemis dan infeksi pada paragraf di atas adalah … .


A. wabah dan masuknya bibit penyakit
B. rawan dan berkurangnya bibit penyakit
C. bahaya dan berkembangnya bibit penyakit
D. ancaman dan keluarnya bibit penyakit
E. risiko dan hilangnya bibit penyakit

2. Bacalah teks berikut dengan saksama!


Kebudayaan tidak mengenal batas seperti negara yang saling berbatasan. Kebudayaan,
seperti juga kebudayaan Melayu, kapan saja dan di mana saja akan tetap ada selama masih
ada orang yang mau menerima. Negara tidak bisa mengklaim sebuah budaya sebagai
miliknya dan memanfaatkannya untuk kepentingan semata. Negara tidak dapat mencegah
penyebaran budaya ke negara lain.

Gagasan pokok paragraf tersebut adalah… .


A. Kebudayaan membatasi akses antarnegara Melayu.
B. Kebudayaan tidak mengenal batas seperti batas antarnegara.
C. Kebudayaan Melayu dibatasi oleh wilayah antarnegara.
D. Kapan saja dan di mana saja budaya harus diterima negara.
E. Kebudayaan merupakan kekayaan suatu negara.

3. Cermatilah paragraf berikut ini!


Banyak orang gemar menyantap buah durian bahkan ada yang sampai lupa membatasi
porsinya. Buah yang terkenal dengan sebutan King of Fruit ini ternyata tidak baik jika
dikonsumsi secara berlebihan. Kandungan gula pada buah durian cukup tinggi sehingga
dapat menyebabkan kadar gula darah dalam tubuh meningkat. Selain itu, kandungan
kalori di dalam durian juga tinggi. Kelebihan asupan kalori dalam tubuh dapat memicu
obesitas dan meningkatkan risiko penyakit jantung dan hipertensi.

Tanggapan yang sesuai berdasarkan paragraf tersebut adalah … .


A. Sebaiknya tidak makan buah durian karena tidak memiliki manfaat bagi tubuh.
B. Sebaiknya tidak berlebihan saat makan durian agar terhindar dari berbagai penyakit.
C. Hindari makan durian karena memicu obesitas, jantung, dan hipertensi.
D. Lebih baik membudidayakan buah durian karena memiliki nilai jual yang tinggi.
E. Buah durian memang enak sehingga banyak orang lupa membatasi porsi makannya.

4. Bacalah teks berikut dengan saksama!


Manusia tidak bisa lepas dari bahasa. Terbukti dalam percakapan sehari-hari,
menyampaikan pendapat dan berkomunikasi tak lepas dari peran bahasa. Bahasa menjadi
ciri identitas suatu bangsa. Melalui bahasa, orang dapat mengidentifikasi kelompok
masyarakat, bahkan dapat mengenali perilaku dan kepribadian masyarakat penuturnya.
Oleh karena itu, masalah kebahasaan tidak terlepas dari kehidupan masyarakat
penuturnya.

Simpulan paragraf tersebut adalah … .


A. Manusia tidak bisa lepas dari bahasa dalam kehidupan sehari-hari.
B. Bahasa digunakan untuk menyampaikan pendapat dan berkomunikasi.
C. Bahasa menjadi ciri identitas suatu bangsa dari perilaku penutur bahasa.
D. Bahasa dapat digunakan untuk mengidentifikasi perilaku penuturnya.
E. Masalah kebahasaan tidak terlepas dari kehidupan masyarakat penuturnya.

5. Cermati paragraf berikut dengan saksama!


(1) Berulang kali ditertibkan, petambang ilegal masih mengeruk pasir di Sungai Progo,
Kulon Progo. (2) Lokasi tambang liar itu sebagian besar berada di kawasan pesisir. (3)
Hampir setiap hari belasan truk bebas hilir mudik mengangkut pasir dari lokasi tambang
ilegal itu. (4) Salah satu lokasi tambang pasir ilegal di kawasan pesisir itu luasnya telah
mencapai tiga kali lapangan sepak bola. (5) Ketiadaan sanksi hukum tegas membuat
petambang kucing-kucingan dengan petugas.

Kalimat yang menyatakan hubungan sebab akibat ditandai dengan nomor … .


A. (1)
B. (2)
C. (3)
D. (4)
E. (5)

6. Cermati paragraf berikut dengan saksama!


(1) Asian Games berpeluang menjadi panggung promosi segala potensi Indonesia. (2)
Jika momentum ini dapat ditangkap, bukan mustahil penyelenggaraan kejuaraan
multicabang ini dapat menggairahkan perekonomian nasional. (3) Geliat ini mulai
terekam dalam sepekan terakhir di sejumlah tempat. (4) Pesta olahraga terbesar se-Asia
ini diperkirakan dihadiri 185.400 tamu mancanegara. (5) Mereka terdiri atas 11.000 atlet,
4.400 ofisial, dan 170.000 suporter asing.

Maksud kalimat ke dua (2) pada teks tersebut adalah … .


A. Momentum penyelenggaraan kejuaraan multicabang olahraga di Indonesia.
B. Penyelenggaraan Asian Games dapat menggaraihkan ekonomi nasional.
C. Kegairahan perekonomian nasional bersamaan dengan Asian Games.
D. Kesiapan menangkap peluang mendapatkan medali multicabang olahraga.
E. Peningkatan perekonomian nasional melalui kejuaraan multicabang olahraga.

7. Bacalah paragraf berikut dengan cermat!


Hujan lebat yang melanda tahun ini merata mengguyur Jakarta dan sekitarnya memicu
banjir di sejumlah lokasi. Banjir terparah terjadi di kawasan Jakarta Selatan. Banjir
terjadi akibat meluapnya saluran dan sungai-sungai, seperti Pesanggrahan, Krukut, dan
Kemang. Bahkan banjir di wilayah Kemang membuat beberapa mobil terendam dan
terpaksa berhenti di tengah jalan. Kondisi serupa juga terjadi di wilayah Gandaria

Tanggapan logis terhadap paragraf tersebut adalah … .


A. Bencana banjir di Jakarta seharusnya dapat diantisipasi sejak dini.
B. Banjir merupakan bencana yang tidak dapat dihindari di Jakarta.
C. Jenis banjir yang melanda Jakarta ini merupakan jenis banjir kiriman.
D. Untuk menjaga agar tidak terkena banjir sebaiknya para penduduk segera mengungsi.
E. Mobil yang terendam banjir sebaiknya segera dibesituakan agar tidak macet di jalan.

8. Cermati paragraf berikut dengan saksama!


(1) Selain banyak memberikan banyak manfaat bagi kesehatan tubuh, bengkoang juga
mempunyai manfaat untuk kecantikan. (2) Banyak produk yang menggunakan
bengkoang sebagai salah satu komposisi di dalam produk kecantikan kulit dan wajah. (3)
Bengkoang mengandung protein nabati yang tinggi. (4) Kandungan nutrisi dalam
bengkoang juga tidak kalah dengan nutrisi pada buah dan sayuran lainnya. (5)
Bengkoang juga memiliki rasa manis dari kandungan glukosa yang ada di dalamnya.

Kalimat yang menyatakan hubungan perbandingan dalam paragraf tersebut adalah … .


A. (1)
B. (2)
C. (3)
D. (4)
E. (5)
9. Cermatilah kutipan teks cerita sejarah berikut!
Meletusnya Perang Dunia (PD) II pada hakikatnya merupakan wujud konkret dari
ambisi dan semangat imperalisme masing-masing negara untuk memperluas daerah
kekuasaannya. Oleh karena itu, pada saat berkobarnya PD II, Indonesia benar-benar
menjadi sasaran perluasan pengaruh kekuasaan Jepang. Bahkan, Indonesia menjadi salah
satu benteng pertahanan Jepang untuk membendung gerak laju kekuatan Sekutu dan
melawan kekuatan Belanda. Setelah berhasil menguasai Indonesia, Jepang mengambil
kebijakan dalam bidang ekonomi yang sering disebut self help. Hasil perekonomian di
Indonesia dijadikan modal untuk mencukupi kebutuhan pemerintahan Jepang yang
sedang berkuasa di Indonesia.

Ringkasan yang sesuai dengan kutipan teks tersebut adalah... .


A. Meletusnya Perang Dunia II merupakan wujud konkret Jepang untuk memperluas
daerah kekuasaannya, termasuk Indonesia yang dijadikan benteng bagi Jepang untuk
membendung laju kekuatan Sekutu dan Belanda.
B. Indonesia dijadikan modal ekonomi bagi Jepang untuk membangun industri yang
sedang dikembangkan untuk memenangkan perang melawan kekuatan militer pihak
Sekutu dan Belanda.
C. Jepang memperluas kekuasaannya pada saat Perang Dunia II dengan menginflasi
kekuatan militer Sekutu dan Belanda melalui kegiatan ekonomi yang dilaksanakan di
Indonesia.
D. Kebijakan ekonomi perang yang diterapkan oleh Sekutu dimodifikasi oleh Jepang untuk
memenangkan Perang Dunia II dengan memanfaatkan Indonesia sebagai benteng
ekonomi.
E. Untuk mencukupi kebutuhan perang di Indonesia, pemerintah Jepang berambisi untuk
menguasai daerah-daerah yang ada di wilayah Indonesia dengan bantuan pihak Sekutu
dan Belanda.

10. Cermatilah teks anekdot berikut dengan saksama!


Setiap sore, duda beranak dua itu selalu melewati rumah bercat hijau. Dandanannya
selalu rapi. Di rumah itu tinggallah keluarga yang memiliki gadis cantik yang menjadi
bunga desa. Duda itu jatuh hati padanya.Suatu sore, duda itu datang ke rumah bercat
hijau untuk meminang anak gadis itu. “Tujuan saya ke rumah Bapak, saya ingin
memperistri anak Bapak.” Katanya dengan sopan. Bapak itu kaget, masak dia tidak
berkaca dulu sebelum meminang anaknya. Kalau kau ingin daun muda, pergilah ke bukit
sebelah di sana banyak pohon yang masih berdaun muda.” Jawab Bapak gadis itu
dengan tegas. Duda itu kaget, “Lho, Pak?” “Maaf, saya tolak, pintu keluar ada di
sebelah sana!” jawab bapak gadis itu dengan gelap mata sambil menunjukkan pintu.

Interpretasi kutipan teks anekdot tersebut adalah... .


A. Seorang duda yang ingin meminang gadis muda.
B. Sorang duda yang ingin mencari daun yang masih muda.
C. Seorang gadis yang ingin dipinang oleh seorang duda.
D. Lamaran seorang perjaka kepada gadis muda.
E. Gadis muda yang menginginkan daun muda.

11. Bacalah penggalan novel berikut ini dengan cermat!


Ketika menginjak kelas 1 SD, ibunya meninggal ketika melahirkan adiknya yang
juga meninggal. Baru kelas 3 SD, dia sudah ditinggal oleh ayahnya yang juga meninggal.
Arai hidup sebatang kara, dan dia dipungut keluarga kami.
(Sang Pemimpi, Andrea Hirata)

Pesan moral yang terkandung dalam penggalan novel tersebut yang paling tepat adalah…
A. Tolonglah sesama yang sangat membutuhkan uluran tangan kita.
B. Belajarlah dengan sungguh-sungguh agar sukses di masa depan.
C. Janganlah menggantungkan hidup kepada orang yang bukan saudara kita.
D. Hargailah setiap pertolongan orang lain kepada kita sekecil apa pun itu.
E. Cintailah setiap anggota keluarga selayaknya mencintai diri sendiri.

12. Cermatilah teks anekdot berikut dengan saksama!


Seorang ibu yang berprofesi sebagai direktur perusahaan terkenal mengajak anaknya
yang berusia 5 tahun dalam acara rapat kantor. Ibu itu berpesan kepada anaknya,

“Nak kalau kamu ingin buang air kecil jangan bilang “kencing” ya, Ibu malu. Biar Ibu
tidak malu Ibu ganti dengan kata “menyanyi” ya.” Anak itu mengangguk setuju. Setiap
kali anak itu bilang menyanyi ibunya langsung mengantarkan ke kamar kecil. Satu
minggu kemudian anak tersebut tinggal bersama neneknya di rumah. Anak itu kemudian
berkata kepada neneknya.

“Nek, aku ingin menyanyi.” Pinta anak tersebut.

“Oh, kalau kamu ingin menyanyi tempelkan saja di telingga nenek”. Jawab neneknya

Ahmad seorang siswa SMK bertanya kepada temannya Sholeh.

“Negara manakah yang memiliki ibu kota lebih dari satu?” tanya Ahmad.

“Negera yang memiliki lebih dari satu ibu kota tidak ada, semua negara itu ibu kotanya
hanya satu, sebagai pusat pemerintahan.” Jawab Sholeh.

“Ada, di dunia ini negara yang memiliki ibu kota lebih dari satu, yaitu Peru. Ibu Kotanya
Lima.

Persamaan kedua teks anekdot tersebut adalah ...


A. Kebiasaan tokoh kedua teks dalam memahami bahasa.
B. Pokok bahasan kedua teks adalah kehidupan sehari-hari.
C. Tokoh utama kedua teks anekdot adalah siswa SMK.
D. Watak tokoh kedua teks adalah cerdik.
E. Objek permasalahan kedua teks adalah pemaknaan bahasa.

13. Cermati penggalan puisi berikut!

Dalam diriku mengalir sungai panjang


Darah namanya
Dalam diriku menggenang telaga
Darah sukma namanya
Dalam diriku meriak gelombang
Sukma hidup namanya
...
(dalam diriku, karya sapardi Djoko Darmono)

Majas yang terdapat dalam penggalan puisi tersebut adalah ...


A. Personofikasi
B. Anafora
C. Metafora
D. Tautologi
E. Hiperbola

14. Cermati teks berikut dengan saksama!


(1)“O, begitu Pak, tidak apa-apa. (2) Saya kan sudah bilang siap menerima apa saja dan
tidak menaruh benci kepada siapa pun. (3) Kualitas manusia itu ditentukan dari akhlak
dan perilakunya, bukan dari bentuk fisiknya. (4) Pengalaman saya di sini cukup banyak,
dan saya kira, sayalah yang harus meminta maaf sekarang. (5) Terlalu merepotkan
selama ini.” Suara Sofyan biasa dan pasti.

Pandangan penulis pada kutipan teks tersebut ditandai dengan nomor … .


A. (1)
B. (2)
C. (3)
D. (4)
E. (5)

15. Bacalah kutipan novel berikut!


Mereka sampai di Pytnitskaya Ulitsa. Mereka menyusuri jalan besar itu terus ke utara.
Sampailah mereka di tepi kanal Moskwa. Ayyas melihat pemandangan yang indah.
Gedung-gedung tua yang tertata rapi. Sungai yang membelah kota. Salju terlihat di
mana-mana. Ia seperti masuk di alam mimpi.

Bumi Cinta: Habiburahman El Shirazy

Tahapan alur yang terdapat pada kutipan tersebut adalah...


A. pengenalan cerita
B. pengungkapan peristiwa
C. menuju konflik
D. puncak konflik
E. penyelesaian (ending)

16. Cermati teks berikut dengan saksama!


Semalam-malam itu keduanya tidak tidur sekejap mata. Rapiah mendengar mertuanya
antara sebentar mengeluh, sedang di dalam dadanya sendiri menyerkap saja apa yang
sedang dirasainya. Bukankah semua itu takhayul belaka, demikian kata Rapiah dalam
hatinya. Suatu pun tak ada bukti yang boleh dipergantungi buat mengekalkan keyakinan
kepada yang buruk-buruk itu. Hanafi baru sebulan meninggalkan rumah. Ia ke Batavia
hendak berobat.

“Salah Asuhan”, Abdoel Moeis

Konflik batin yang dialami Rapiah disebabkan karena … .


A. Tidak dapat tidur dalam semalam.
B. Keluhan mertuanya dan perasaan tentang takhayul.
C. Kata hati Rapiah mengenai takhayul yang dialami.
D. Tidak dapat menemukan bukti tentang takhayul.
E. Kepergian Hanafi ke Batavia untuk berobat.

17. Bacalah kutipan cerpen berikut ini!


Nina pun segera keluar kelas. Tangga demi tangga ia telusuri hingga akhirnya ia sampai
di tempat parkir. Sesampainya di tempat itu. Nina benar-benar kaget karena sepedanya
tidak ada.
“Aduh, sepedaku mana?”
Latar tempat kutipan cerpen tersebut adalah... .
A. di ruang kelas
B. di kantin sekolah
C. di tempat parkir
D. di halaman sekolah
E. di aula sekolah

18. Cermati teks berikut dengan saksama!


Seperti biasanya, pada hari Minggu sore Bu Satria menyempatkan diri untuk melihat-
lihat tamannya yang kelihatan sangat asri. Ia membersihkan daun-daun atau bunga-bunga
yang sudah layu. Ia juga melihat tanaman yang ditanam di pot gantung. Pot-pot gantung
itu tidak seluruhnya dibeli dari toko. Kebanyakan pot-pot gantung tersebut terbuat dari
barang-barang bekas. Selain dari kaleng bekas tempat kue atau susu, juga dari bekas
tempat nasi, wajan, dan panci yang tidak digunakan lagi.

“Hati yang Tegar”, karya Kamalasari

Kaitan cerita dengan kehidupan sehari-hari pada saat ini adalah … .


A. Melihat bunga yang asri dan indah setiap hari Minggu.
B. Menggunakan barang bekas untuk menanam tanaman hias.
C. Menggunakan panci sebagai tempat menanam bunga.
D. Menghias rumah dengan gantungan pot bunga di teras.
E. Membersihkan daun dan bunga dari tanaman di dalam pot.

19. Bacalah kutipan puisi berikut!


Kepada-Mu aku mohon ampun
Langkahku telah kulanggar
Aku datang ke tempat yang kau larang
Aku dapatkan semua ini tak halal

Tema puisi tersebut adalah… .


A. kekecewaan kepada kekasih
B. penyesalan kepada kekasih
C. pertobatan kepada Tuhan
D. kekecewaan kepada Tuhan
E. keputusasaan kepada Tuhan

20. Cermati kutipan puisi berikut!


Kami bicara padamu dalam hening di malam sepi


Jika dada rasa hampa dan jam dinding yang berdetak
Kenang, kenanglah kami
Teruskan, teruskanlah jiwa kami
Menjaga Bung Karno
Menjaga Bung Hatta
Menjaga Bung Syahrir
Kami sekarang mayat
Beri kami arti
Berjagalah terus di garis batas pernyataan dan impian

Chairil Anwar

Pesan yang tersirat dari kutipan puisi tersebut adalah … .


A. Mendengarkan pembicaraan para penyair di malam hari.
B. Mencermati waktu seperti jam dinding yang berdetak.
C. Mengenang perjuangan para pahlawan kemerdekaan.
D. Melanjutkan cita-cita para pahlawan yang telah gugur.
E. Mengartikan pernyataan dan impian para pahlawan.

21. Bacalah penggalan naskah drama berikut!


Adegan Ponirah dan Marini dengan menggendong bakul dan mengenakan topi caping.
Marni : Pon … Ponirah!
Ponirah : Ada apa?
Marni : Aku melihat sepintas bayangan orang di sana!
Ponirah : Tenang saja!
Marni : Tenang… Tenang! Tenang bagaimana? Kalau musuh?
Ponirah : Musuh? Marni, kita ini jualan buah dan tidak punya musuh. Kita harus yakin,
yang berani bergerak di malam hari hanya TNI. Ayo jalan!
Marni : Tapi bulu kudukku berdiri.
Ponirah : Makanya jangan di sini, ayo terus jalan!

Sikap Marni yang tergambar dalam adegan drama tersebut adalah… .


A. berani
B. terburu-buru
C. tenang
D. ragu
E. cemas
22. Perhatikan data buku berikut ini!
Buku Panduan Keluarga Memilih dan Menggunakan Obat karya Rahayu Widodo,
S.Si.,Apt. berisi tentang pengenalan akan obat, prinsip-prinsip dasar memilih, dan
menggunakan obat untuk diri sendiri dan aspek sosialnya. Buku ini menjadi panduan
yang jelas, ringkas, dan dilengkapi dengan arti istilah medis.

Kalimat resensi yang menyatakan keunggulan sesuai dengan data tersebut adalah... .
A.Rahayu Widodo menulis buku tentang bagaimana memilih dan menggunakan obat untuk
diri sendiri. Buku ini dilengkapi dengan daftar istilah beserta artinya.
B.Penulis, yang seorang apoteker, menyampaikan penjelasan dengan bahasa yang
sederhana dan mudah dimengerti, juga melengkapi buku ini dengan istilah-istilah medis.
C. Telah hadir sebuah buku panduan bagaimana memilih dan menggunakan obat yang
ditulis oleh seorang dokter dengan bahasa yang mudah dipahami dan ada daftar arti
istilah medis.
D. Dengan adanya penjelasan istilah-istilah medis dalam buku panduan ini, akan
memudahkan pembaca dan memahami isi buku, apalagi penulisannya adalah dokter
yang berpengalaman.
E.Buku ini berisi tentang pengenalan akan obat, prinsip-prinsip dasar memilih, dan
menggunakan obat.

23. Bacalah penggalan novel berikut!


Ia seorang gadis tani, anak peladang desa, yang hidupnya selalu dalam serba kekurangan.
Ayahnya meninggal di Merak dalam pekerjaan romusha ciptaan Jepang, dan ibunya
sudah berpulang sehari sesudah terjadi pembakaran kampung karena penyakit tua dan
karena menderita “putus asa”. Sebagai gadis yang menginjak dewasa, Warsiah pernah
mempunyai pujaan hati, Masri namanya, teman sekampung halaman, seorang pemuda
yang cakap tampan, pernah masuk “peta” di masa Jepang dan dalam pemerintahan
Republik, jadi pemimpin rombongan BKR kampung yang sangat aktif. Masri seorang
pemuda yang bercita-cita tinggi. Pernah berkata di hadapan kekasihnya, bahwa ia tak kan
beristri dulu kalau Indonesia belum merdeka.

Ringkasan dari penggalan novel tersebut adalah….


A. Seorang gadis tani yang mendambakan kekasih bernama Masri tetapi Masri
mengkhianati cinta gadis itu.
B. Ayah Warsiah meninggal di Merak dalam pekerjaan romusha ciptaan Jepang dan
ibunya sehari sesudah terjadi pembakaran kampong karena penyakit tua dan karena
menderita “putus asa”.
C. Warsiah, seorang gadis tani yang pernah mempunyai kekasih bernama Masri, seorang
pemuda yang rela berjuang demi Indonesia Merdeka.
D. Kegigihan Masri melawan penjajah Jepang dan meninggalkan keluarganya.
E. Orang tua yang putus asa karena hidup serba kekurangan.

24. Cermati paragraf berikut!


Gempa tektonik yang mengguncang Kabupaten Nabire Provinsi Papua. […] Gempa
berkekuatan 6,91 skala richter itu menyebabkan sedikitnya 26 korban tewas, 34 luka
berat, dan 66 orang lainnya luka ringan. Guncangannya yang keras itu juga menyebabkan
tanah longsor dan meretakkan landasan pacu di bandara setempat.

Kalimat yang tepat untuk melengkapi paragraf tersebut adalah… .


A. Guncangan keras yang susul-menyusul hingga enam kali itu terjadi selama dua jam.
B. Selain korban jiwa, masih ditambah kerusakan materi yang belum bisa diprediksi
seberapa besar kerugiannya.
C. Gempa yang terjadi di wilayah Indonesia berbeda dengan gempa yang terjadi di negara
lain.
D. Tugas Tim Basarnas dalam setiap bencana alam antara lain mengevakuasi para korban.
E. Selain pemerintah daerah, masyarakat umum juga memberi bantuan untuk para korban

25. Bacalah kalimat berikut ini!


Muhammad Fatchu Rohman merupakan bek sayap Tim Nasional Indonesia U-19 yang
berhasil mengangkat [....] keluarga melalui sepak bola.

Kata yang tepat untuk melengkapi kalimat tersebut adalah… .


A. derajat
B. harkat
C. kedudukan
D. pangkat
E. jabatan

26. Cermatilah paragraf rumpang berikut ini!


Datangnya musim penghujan ternyata mendongkrak penjualan produk-produk
elektronik. Produk yang paling menjadi incaran adalah mesin cuci. Ibu-ibu rumah tangga
dan pengusaha jasa laundry sangat terbantu dengan adanya mesin cuci. Minimnya panas
matahari pada musim penghujan membuat pakaian yang dicuci lebih lama untuk dapat
menjadi kering. [...]

Kalimat yang tepat untuk mengisi bagian rumpang dalam paragraf tersebut adalah ... .
A. Dengan mesin cuci berpengering otomatis, proses pengeringan menjadi lebih cepat.
B. Ibu-ibu rumah tangga sudah terbiasa menjemur di bawah sinar matahari langsung.
C. Pada musim penghujan mengakibatkan harga barang-barang elektronik mahal.
D. Para pengusaha jasa laundry mengincar berbagai macam produk elektronik.
E. Dengan menjemur di sinar matahari langsung lebih menguntungkan masyarakat.

27. Cermati paragraf eksposisi yang rumpang berikut!


Makanan tradisional lebih baik daripada makanan cepat saji. [ … ] Makanan cepat saji
miskin serat banyak lemak dan gula. Akibatnya terjadi obesitas dan peningkatan
kolesterol dalam darah.

Kalimat penjelas yang tepat untuk melengkapi paragraf eksposisi yang rumpang tersebut
adalah… .
A. Makanan cepat saji lebih cepat penyajiannya dan banyak penggemarnya.
B. Makanan tradisional banyak mengandung serat yang berguna dan melindungi diri dari
kanker.
C. Makanan cepat saji sangat tepat untuk zaman sekarang karena singkat dalam
penyajiannya.
D. Makanan tradisional adalah makanan sehat namun kurang diminati.
E. Makanan merupakan kebutuhan semua orang yang ingin hidup sehat.

28. Bacalah pantun berikut!


Menjahit baju pakailah benang
benang rajut diulang-ulang
[...]
kecuali datang untuk meminang

Larik yang tepat untuk melengkapi pantun tersebut adalah... .


A. Besok pagi aku akan datang
B. Saya harap engakau akan datang
C. Tiada lain maksudku datang
D. Percayalah aku akan datang
E. Sudilah kasihku untuk datang

29. Bacalah teks ulasan berikut dengan saksama!


Buku Remaja Membangun Kepribadian menarik untuk dibaca semua kalangan. Buku
Remaja Membangun Kepribadian memiliki ciri khas dibandingkan buku lain psikologi
remaja. Gaya bahasa penulis mudah dipahami oleh pembaca. [...] Sebagian besar buku
psikologi remaja menyampaikan gagasan secara berbelit-belit.

Kalimat yang tepat untuk melengkapi resensi rumpang tersebut adalah ....
A. Berbagai jenis kecerdasan dan kemampuan dalam diri manusia dapat digunakan untuk
mengembangkan diri.
B. Tidak hanya itu, pembahasan dalam Buku Remaja Membangun Kepribadian singkat
sehingga pembaca mudah memahami maksudnya.
C. Dalam buku ini dibahas visi dan misi yang harus ada dalam setiap diri manusia agar
dapat mengembangkan potensinya.
D. Gambar ilustrasi yang digunakan masih kurang sehingga memungkinkan terjadinya
kepenatan pembaca.
E. Buku Remaja Membangun Kepribadian sebaiknya mengangkat berbagai peristiwa yang
dekat dengan remaja saat ini.

30. Bacalah teks negosiasi berikut ini dengan saksama!


Penjual : [“...”]

Pembeli : “Selamat sore.”

Penjual : “Ibu mau beli apa?”

Pembeli : “Ada kain tenun Lombok?”

Penjual : “Ada Bu, kami punya beberapa koleksi tenun Lombok.”

Pembeli : “Ada berapa wananya ya?”

Penjual : “Bermacam-macam Bu, mari saya antar agar Ibu bisa memilih sesuai
dengan warna yang diinginkan.”

Pembeli : “Tolong carikan yang berwarna hijau muda ya?”

Penjual : “Ini, sudah saya pilihkan sesuai dengan keinginan Ibu. Bahannya halus,
warnanya hijau muda dan sangat elegan.”

Pembeli : “Jika beli banyak bisa dapat potongan harga?”

Penjual : “Untuk saat ini, sudah harga kami yang paling murah, Ibu.”

Pembeli : “Kalau begitu saya ingin lima belas potong.”

Kalimat yag tepat untuk melengkapi paragraf pembuka yang rumpang adalah ....
A. Selamat sore.
B. Selamat berjumpa Ibu
C. Hallo selamat sore .
D. Selamat sore Ibu.
E. Apa kabar hari ini Ibu?

31. Cermatilah teks cerpen berikut!


1) Dan perkedelnya juga terasa lebih konkret karena banyak dagingnya.
2) Hanya dua macam lauk itu, tetapi Sri sendiri yang sengaja memasaknya.
3) Hidangan buka puasa sore itu terdiri dari agar-agar dingin dengan sirup merah yang
manis sekali, risoles, dan es teh manis.
4) Kemudian untuk makan malam sup kacang merah dengan daging sengkel dengan
isian daging yang cukup tebal.
5) Maka kaldu sup itu juga lebih terasa kental dan mirasa, kacang merahnya juga
gemuk-gemuk merah.
(Umar Kayam, Menjelang Lebaran)

Urutan yang tepat kalimat-kalimat kutipan teks cerita pendek tersebut adalah ... .
A. (2), (4), (5), (1), (3)
B. (3), (2), (4), (5), (1)
C. (3), (4), (2), (5), (1)
D. (3), (5), (1), (4), (2)
E. (4), (2), (5), (1), (3)

32. Cermatilah kalimat-kalimat berikut!


(1) Jika salah memilih teman, kita akan gagal dalam menjalin persahabatan.
(2) Sungguh hal itu hanya dilakukan dalam batas-batas yang wajar.
(3) Namun, bukan berarti dalam memilih teman harus meragukan semua orang.
(4) Memilih teman perlu dilakukan dengan terencana, tidak spontan.
(5) Sebab teman adalah orang yang paling dekat dengan kita.

Kalimat-kalimat tersebut dapat menjadi sebuah paragraf deduktif apabila disusun dengan
urutan... .
A. (1), (3), (4), (2), (5)
B. (2), (3), (4), (1), (5)
C. (3), (4), (1), (5), (2)
D. (4), (5), (1), (3), (2)
E. (5), (4), (2), (3), (1)

33. Cermati teks berikut!


(1) Suka duka ada dalam kehidupan ini. (2) Kita ingin usaha kita maju dan dikenal
masyarakat luas. (3) Namun tidak jarang kita dalam usaha harus berhadapan dengan
orang-orang yang menipu. (4) Hingga, usaha kita menjadi bangkrut. (5) Namun, kita
harus tetap tabah dan tegar menghadapi hal ini.

Kalimat yang di dalamnya terdapat kata bersinonim ditandai dengan nomor… .


A. (1)
B. (2)
C. (3)
D. (4)
E. (5)

34. Bacalah paragraf berikut dengan saksama!

Kabar burung santer terdengar bahwa di kampung Salimin akan dibangun masjid
terbesar di dunia. Salimin mendengar dari tetangga-tetangganya yang bergunjing
tentang masjid itu. Masjid itu nanti kubahnya saja akan berlapis emas, lampu-lampunya
terbuat dari kristal, pilar-pilarnya dilapisi marmer terbaik dari Eropa. Bahkan,
arsitekturnya modern berkiblat Timur Tengah.
Makna ungkapan kabar burung yang terdapat dalam penggalan cerpen tersebut adalah... .
A. Kabar nyata
B. kabar bohong
C. kabar yang sudah pasti
D. kabar menggembirakan
E. kabar yang menyedihkan

35. Cermatilah grafik berikut dengan saksama!

GRAFIK PESERTA EKSTRAKURIKULER


SMK BAKTI UTAMA TAHUN 2019
100
80
60
40
20
0
Karawitan Tari Basket Jurnalistik
laki-laki perempuan

Simpulan isi grafik tersebut adalah ... .


A. Karawitan sangat diminati oleh siswa SMK Bakti Utama
B. Jurnalistik kurang diminati oleh siswa SMK Bakti Utama
C. Setiap jenis ekstrakurikuler didominasi oleh siswa perempuan.
D. Tidak semua siswa mengikuti ekstrakurikuler di sekolahnya.
E. Jumlah peserta karawitan sama dengan jumlah peserta jurnalistik.

36. Cermati tabel berikut!

Pernyataan yang sesuai dengan tabel tersebut adalah … .

A. Kandungan lemak jagung lebih banyak dari dedak padi.


B. Dedak padi adalah bahan pakan yang paling tidak diminati.
C. Kadar energi ampas tahu (fermentasi) lebih banyak dari pada jagung.
D. Bahan pakan yang kandungan energinya paling tinggi adalah jagung.
E. Kandungan gizi dalam tepung ikan selalu lebih tinggi dari pada jagung.

37. Cermati paragraf berikut!


(1) Kelapa adalah anggota tunggal dalam marga Cocos dari suku aren-arenan atau
Arecaceae. (2) Arti kata kelapa dapat merujuk pada keseluruhan pohon kelapa, biji,
atau buah, yang secara botani adalah pohon berbuah, bukan pohon kacang-kacangan.
(3) Istilah ini berasal dari Portugis dan Spanyol abad ke-16, coco yang berarti
“kepala” atau “tengkorak” setelah tiga lekukan pada tempurung kelapa yang
menyerupai fitur wajah. (4) Tumbuhan ini dimanfaatkan hampir semua bagiannya
oleh manusia sehingga dianggap sebagai tumbuhan serbaguna, terutama bagi
masyarakat pesisir. (5) Batang pohon kelapa dimanfaatkan manusia untuk membuat
kerangka rumah.

Kalimat opini dalam paragraf di atas terdapat dalam kalimat nomor... .


A. (1)
B. (2)
C. (3)
D. (4)
E. (5)

38. Cermati paragraf berikut!


(1) Internet saat ini sudah sangat melekat dalam kehidupan sehari-hari, terutama di
kalangan remaja. (2) Memang internet cukup membantu dalam menyelesaikan banyak
pekerjaan. (3) Akan tetapi, ternyata internet pun tidak terlepas dari hal-hal negatif yang
dapat berdampak buruk. (4) Maka dari itu, usia remaja merupakan saat-saat labil
sehingga masih sering terseret arus. (5) Oleh karena itu, penelitian mengenai bahaya
internet bagi remaja penting untuk dilakukan.

Konjungsi antarkalimat yang tidak tepat terdapat pada nomor... .


A. (1)
B. (2)
C. (3)
D. (4)
E. (5)

39. Cermati paragraf berikut dengan saksama!

1) Sebagai warga negara Indonesia, sudah semestinya kita mengenal tokoh-tokoh


pahlawan yang berjuang dan berjasa dalam membangun negara kita, Indonesia. (2) Di
dalam sinetron yang ditayangkan di televisi tokoh yang berjiwa pahlawan tidak
digambarkan menurut semestinya. (3) Kita dapat  mengenal tokoh pahlawan melalui
berita atau cerita guru, buku-buku sejarah Indonesia atau biografinya.(4) Mengetahui
tokoh dan perjuangan dapat memberikan keteladanan kepada kita. (5) Di samping itu,
untuk menambah rasa patriotik.

Kalimat sumbang pada paragraf tersebut ditandai dengan nomor… .


A. (1)
B. (2)
C. (3)
D. (4)
E. (5)

40. Cermati paragraf berikut!


Pentingnya penghijauan untuk mengurangi peristiwa [...], penghijauan sangat penting bagi
sekolah, selain menyejukan udara disekitanya, dapat juga memperindah pemandangan.
Penghijauan tidak hanya menanam pohon, dengan membersihkan setiap ruangan dan
lingkungan sekitar sekolah, serta membuang sampah pada tempatnya juga termasuk cara
[...] untuk mewujudkan penghijauan. Salah satu cara kecil yang dilakukan adalah
menyediakan tong sampah di setiap ruangan kelas dan lingkungan sekolah.
Istilah yang tepat untuk melengkapi bagian rumpang pada kalimat dalam paragraf tersebut
adalah … .
A. reboisasi dan efektif
B. kebakaran dan instan
C. green school dan efektif
D. global warming dan instan
E. global warming dan efektif

41. Cermati penggalan teks eksplanasi berikut!


Uap air yang menjadi cair akibat kondensasi ini terus berkumpul [...] langit hingga pada
akhirnya jatuh [...] bumi akibat gaya gravitasi. Proses jatuhnya butiran air ini disebut
presipitasi. Selanjutnya air yang turun ini disebut dengan hujan. Hujan akan turun dan
mengalir [...] tempat yang tinggi ke tempat yang lebih rendah.

Kata depan (preposisi) yang tepat untuk melengkapi bagian rumpang teks eksplanasi tersebut
adalah ....
A. pada, di, dari
B. ke, di, pada
C. ke, ke, dari
D. di, ke, dari
E. dari, ke, di

42. Cermati paragraf berikut dengan saksama!


Akhir-akhir ini, demonstrasi kerap terjadi hampir setiap waktu dan terjadi di berbagai
tempat.[...]. Menanggapi fenomena tersebut, seorang kepala daerah menyatakan bahwa
penyebab demonstrasi dan anarkisme tidak lain adalah faktor laparnya masyarakat. Lantas
ia mencontohkan rakyat Malaysia dan Brunei yang adem ayem, lantaran kesejahteraan
mereka terpenuhi maka demonstrasi di negara-negara itu jarang terjadi.

Kalimat yang tepat untuk melengkapi bagian rumpang pada teks tersebut adalah … .
A. Bahkan, demonstrasi sudah menjadi fenomena yang lumrah di tengah-tengah masyarakat
kita.
B. Mereka berpendirian bahwa demonstrasi yang biasa terjadi murni untuk memperjuangkan
kebenaran.
C. Namun demikian, umumnya demonstrasi massa justru lebih didasari oleh kebutuhan
tingkatan akhir.
D. Masyarakat berdemonstrasi karena membutuhkan pengakuan dari pemerintah ataupun
pihak-pihak lain.
E. Demonstrasi massa tidak selalu disebabkan oleh urusan perut, bahkan banyak peristiwa
yang tidak didasari motif itu.

43. Cermati penggalan teks rumpang berikut!


Keterbatasan anggaran pemerintah dan kurangnya [...] antarlembaga riset mulai
diatasi Dewan Riset Nasional. Untuk mencapai keterpaduan, harus dilakukan [...]
peraturan yang berkaitan dengan riset itu. Dengan demikian, akan terbentuk [...] satu
arah dalam program ataupun penganggaran

Kata yang tepat untuk melengkapi teks rumpang tersebut adalah....


A. sinergi, harmonisasi, kebijakan
B. sinergi, harmonisasi, hukum
C. atensi, sinkronisasi, hukum
D. energi, harmonisasi, proyek
E. anergi, efisiensi, harapan

44. Cermati penggalan surat berikut!


Berdasar iklan lowongan pekerjaan yang dimuat dalam Harian Banjarmasin post, tanggal
17 januari 2020, saya sangat mengharapkan diberi kesempatan untuk dapat bekerja di
perusahaan yang Bpk/Ibu pimpin.

Perbaikan kalimat pembuka surat lamaran pekerjaan berikut yang benar adalah ....

A. Berdasarkan iklan lowongan pekerjaan yang dimuat di Harian Banjarmasin Post,


tanggal 17 januari 2020, saya yang bertanda tangan di bawah ini, ingin melamar
pekerjaan di perusahaan Bapak/Ibu.
B. Berdasarkan iklan lowongan pekerjaan yang dimuat dalam Harian Banjarmasin Post,
17 Januari 2020, saya bermaksud melamar pekerjaan untuk mengisi posisi yang
ditawarkan oleh perusahaan yang Bapak/Ibu pimpin.
C. Sehubungan dengan iklan lowongan pekerjaan yang dimuat di Harian Banjarmasin
Post, tanggal 17 Januari 2020, saya berinisiatif mengajukan lamaran pekerjaan di
perusahaan yang Bapak/Ibu pimpin karena kualifikasi yang Bapak/Ibu syaratkan
sangat sesuai dengan kualifikasi yang saya miliki.
D. Mendengar iklan kebutuhan tenaga kerja yang dimuat dalam Harian Banjarmasin
Post, tanggal 17 Januari 2020, saya tertarik untuk mengisi lowongan tersebut.
E. Saya telah membaca iklan di Harian Banjarmasin Post, tanggal 17 januari 2020,
tentang lowongan pekerjaan di perusahaan Bapak/Ibu dan saya tertarik untuk melamar
pekerjaan tersebut.

45. Cermati penggalan esai berikut!


Pendidikan 4.0 bagaimanapun membuka kesempatan luas bagi peserta didik kejuruan
atau fokasi dalam mengembangkan kreatipitas dan peningkatan daya inovasinya. Oleh
karena itu, sudah tepat apabila pemerintah melalui berbagai regulasi dan pendekatan,
berihtiar untuk menyambungkannya dengan dunia usaha dan dunia industri (DUDI).

Perbaikan kata yang bercetak miring pada paragraf tersebut adalah … .


A. vokasi, kreatifitas, berikhtiar
B. pokasi, kreativitas, beriktiar
C. fokasi, kreatifitas, beriktiar
D. pokasi, kreativitas, berikhtiar
E. vokasi, kreativitas, berikhtiar

46. Cermati paragraf berikut dengan saksama!


Kasus kematian paus yang ditemukan di wilayah Taman Nasional (TN) Wakatobi,
Sulawesi Tenggara, bisa menjadi moment seluruh pihak untuk lebih keras menangani isu
sampah. Kasus tersebut menjadi pengingat bahwa penyelamatan ekosistem laut dari
ancaman sampah plastik mendesak dilakukan. Sekarang saatnya semua pihak
memrioritaskan penanganan terhadap sampah plastik agar tidak ada lagi kasus kematian
ekosistem laut yang lain akibat sampah plastik. Salah satunya adalah dengan menindak
tegas aktifitas pembuangan sampah plastik di laut.

Kata yang tidak sesuai dengan EYD pada paragraf tersebut adalah ... .
A. menindak, ekosistem
B. penyelamatan, aktifitas
C. penanganan, ekosistem
D. menangani, moment
E. memrioritaskan, aktifitas

47. Cermati alamat surat berikut!


Kepada
Yth: Direktur P.T. Mutiara Abadi
Jln Bhayangkara No 31,
Kulon Progo.

Perbaikan penulisan alamat surat yang benar adalah... .


A. Kepada
Yth: Direktur P.T. Mutiara Abadi
Jln Bhayangkara No 31,
Kulon Progo.
B. Kepada
Yth. Direktur P.T. Mutiara Abadi
Jln Bhayangkara No 31
Kulon Progo
C. Yth. Direktur PT Mutiara Abadi
Jalan Bhayangkara 31
Kulon Progo
D. Yth. Direktur P.T. Mutiara Abadi
Jalan Bhayangkara 31
Kulon Progo
E. Yth. Direktur PT Mutiara Abadi
Jalan Bhayangkara 31,
Kulon Progo
48. Cermati paragraf berikut dengan saksama!
Burhanuddin menanggapi soal hubungan keluarganya dengan Politikus PDI Perjuangan,
TB Hasanuddin. Adapun Burhanuddin merupakan adik kandung dari Hasanuddin. "[...]
sudah diperintahkan profesional. Tidak ada saudara, tidak ada apa-apa," kata
Burhanuddin di Kompleks Kejaksaan Agung, [...] mengaku tidak mempermasalahkan
hal tersebut. Burhanuddin pun menegaskan bahwa dirinya bukan orang partai dan akan
bekerja profesional.

Kata ganti (pronomina) yang tepat untuk melengkapi bagian rumpang tersebut adalah … .
A. kami, ia
B. kita, dia
C. mereka, ia
D. kami, saya
E. mereka, saya

49. Cermati paragraf berikut!


Mengonsumsi buah mangga secara teratur, ternyata memberikan manfaat yang luar biasa
pada tubuh. Di samping rasanya yang manis dan segar, antioksidan yang terdapat pada
buah mangga mampu menyehatkan pencernaan, mencegah timbulnya kanker,
menyembuhkan panas dalam, dan mampu mereduksi kandungan kolesterol dalam darah.
[…]

Kalimat simpulan yang tepat untuk melengkapi paragraf tersebut adalah ... .
A. Oleh sebab itu, bagi penderita kolesterol jangan lupa mengonsumsi buah mangga setiap
hari
B. Oleh karena itu, setiap keluarga wajib menanam pohon mangga demi masa depan anak-
anak.
C. Dengan demikian, biasakanlah mengonsumsi buah mangga agar kebugaran tubuh tetap
terjaga.
D. Jadi, gerakan mengonsumsi buah mangga perlu diwujudkan oleh seluruh masyarakat
Indonesia.
E. Dengan demikian, masyarakat harus menyadari bahwa buah mangga adalah buah yang
luar biasa.

50. Cermati penulisan catatan kaki berikut!


Rachmat Djoko Pradopo. Pengkajian Puisi (Gadjah Mada University Press, Yogyakarta:
2002), hal 54.
Perbaikan penulisan catatan kaki tersebut adalah … .
A. Pradopo, Rachmat Djoko. Pengkajian Puisi. 2002. Yogyakarta: Gadjah Mada
University Press. hlm. 54.
B. Rachmat Djoko Pradopo, Pengkajian Puisi (Yogyakarta: Gadjah Mada University
Press, 2002), hlm. 54.
C. Pradopo, Rachmat Djoko, Pengkajian Puisi, Yogyakarta; Gadjah Mada University
Press, 2002, hlm. 54.
D. Rachmat Djoko Pradopo. Pengkajian Puisi (Yogyakarta: Gadjah Mada University
Press. 2002) hlm. 54.
E. Rachmat Djoko Pradopo, Pengkajian Puisi, (Gadjah Mada University Press:
Yogyakarta, 2002) hal. 54.

Anda mungkin juga menyukai